Nephro/Uro/GI Flashcards

1
Q

How does fluid move out of the glomerulus?

A

Net glomeluar pressure

Regulated by afferent and efferent tubules

How well did you know this?
1
Not at all
2
3
4
5
Perfectly
2
Q

Where does fluid go when it leaves the afferent and efferent vessels?

A

Bowmans space

How well did you know this?
1
Not at all
2
3
4
5
Perfectly
3
Q

After leaving Bowman’s space, where does the urine go?

A

Proximal convoluted tubule

Function: bulk isosmotic reabsoprtion of fluid (70%)

ATP dependent

How well did you know this?
1
Not at all
2
3
4
5
Perfectly
4
Q

What is the function of the loop of henle?

A

Elegant counter current type system to concentrate urine

How well did you know this?
1
Not at all
2
3
4
5
Perfectly
5
Q

What is the function of the distal convoluted tubule?

A

Fine tining of the ultrafiltrate

ADH action

How well did you know this?
1
Not at all
2
3
4
5
Perfectly
6
Q

What are the three components of the glomerular basement membrane?

A
  1. Endothelium
  2. Basement membrane
  3. Podocytes
How well did you know this?
1
Not at all
2
3
4
5
Perfectly
7
Q

What is GFR?

A

Glomerular filtration rate

Basically how much blood is effectively “cleaned’ by the kidneys per minute.

Caculated by creatinine clearance or Schwartz formula

How well did you know this?
1
Not at all
2
3
4
5
Perfectly
8
Q

What is the defintion of nephrotic range proteinuria?

A

>40 mg/m2/hr

How well did you know this?
1
Not at all
2
3
4
5
Perfectly
9
Q

When would you order a DSMA scan?

A

“Snapshot” of the kidneys”

Scars, cortical mass

Differential function

How well did you know this?
1
Not at all
2
3
4
5
Perfectly
10
Q

What are the types of dynamic renal scans?

A

DTPA, MAG3

Uptake, excretion, differential function

How well did you know this?
1
Not at all
2
3
4
5
Perfectly
11
Q

When should you order a VCUG?

A

Male with UTI and suspicion of PUV

AbN AUS

TO R/o VUR

How well did you know this?
1
Not at all
2
3
4
5
Perfectly
12
Q

What can mimic hematuria?

A

Dyes (beets, rifampin, food dye)

Hemoglobinuria

Myoglobinuria

Menstruation

Factitious

How well did you know this?
1
Not at all
2
3
4
5
Perfectly
13
Q

What are broad classifications of hematuria?

A
  1. Vascular
  2. Glomerular (MOST COMMON)
  3. Tubulointerstitial
  4. Lower urinary tract
  5. coagulopathy
How well did you know this?
1
Not at all
2
3
4
5
Perfectly
14
Q

What are the proliferative causes of glomerular causes of hematuria? (Immune complex mediated)

A
  1. Ig A Nephroapthy
  2. Post infectious GN
  3. HSP
  4. MPGN

Chronic infection

RPGN

How well did you know this?
1
Not at all
2
3
4
5
Perfectly
15
Q

What are the characteristics of proliferative glomerular causes of hematuria?

A

Immune mediated (immune complexes or IG deposition into kdiney)

“Crescents” SLE, Goodpastures, Wegener’s

VERY POOR PROGNOSIS

How well did you know this?
1
Not at all
2
3
4
5
Perfectly
16
Q

What are the characteristics of nonproliferative glomerular hematuria?

A
  • Abnormal GMB
  • No immune deposition
  • Alports: abN type 4 collagen (kidney, cochlear, eye)
  • Thin GBM: thin
How well did you know this?
1
Not at all
2
3
4
5
Perfectly
17
Q

What are the tubulointerstitial causes of hematuria?

A
  1. ATN
  2. Tubulointersitital nephritis
  3. ADPKD
How well did you know this?
1
Not at all
2
3
4
5
Perfectly
18
Q

What are the most common causes of hematuria?

A
  1. Post infectious GN
  2. Alports
  3. HSP
  4. HUS
  5. IgA N
How well did you know this?
1
Not at all
2
3
4
5
Perfectly
19
Q

What are the characteristics of post infectious GN?

A
  • Immune reaction to GBM after skin/throat strep infection
  • Nephritic pictures: macrohematuria, edema, HTN, proteinuria, renal failure
  • Proliferative glomerular
  • +ASOT, DECREASED C3, normal C4
  • Usually self-limiting
How well did you know this?
1
Not at all
2
3
4
5
Perfectly
20
Q

What are the characteristics of Alport’s syndrome?

A

Hereditary nephritis

X-linked, female carrier or AR, AD

Type 4 collagen (Cochlear/Eyes/Renal)

Amount of proteinuria is important

Most require transplant

How well did you know this?
1
Not at all
2
3
4
5
Perfectly
21
Q

What are the characteristics of henoch-schonlein purpura?

A
  • Leukocytoclastic vasculitis (kidney, gut, skin, joints)
  • Biopsy = to IgA

Rx symptomatic

Divided into D- and D+

How well did you know this?
1
Not at all
2
3
4
5
Perfectly
22
Q

What is the classic triad of HUS?

A
  1. Anemia (Coombs negative)
  2. Thrombocytopenia
  3. Renal failure

10% will develop HUS from E COli 0157:H7

How well did you know this?
1
Not at all
2
3
4
5
Perfectly
23
Q

What is the treatment of HUS?

A

Medical RX of renal failure

Dialysis if necessary

Plasma exchange?

NO antibiotics

NO platelets unless absolutely necessary

PRBC as necessary

How well did you know this?
1
Not at all
2
3
4
5
Perfectly
24
Q

What are the characteristics of IgA nephropathy?

A

Most common cause of gross

Hematuria with URTI and illness

10-20% will develop ESRD

Less favourable: proteinuria, male gender, lack of gross hematuria

Careful f/u

How well did you know this?
1
Not at all
2
3
4
5
Perfectly
25
Q

What are the categories of renal stones?

A

Calcium oxalate/phosphate

Struvite (infectious stones)

Uric acid

Most treatment: high fluid intake, pain control

How well did you know this?
1
Not at all
2
3
4
5
Perfectly
26
Q

What are the characteristics of MPGN?

A
  • Biopsy diagnosis
  • Primary in children
  • Infection, autoimmune
How well did you know this?
1
Not at all
2
3
4
5
Perfectly
27
Q

If patient with polyhydramnios presented with hypoK, alkalosis, polyuria, hypercalciuria, what would they have?

A

Barter Syndrome

Like giving someone a loop diuretic

Can be associated with growth and MR

How well did you know this?
1
Not at all
2
3
4
5
Perfectly
28
Q

If a patient presented with hypoK, alkalosis, hypoMg, Ca in late childhood, what syndrome would they have?

A

Gitelman syndrome

Like giving too much thiazide

How well did you know this?
1
Not at all
2
3
4
5
Perfectly
29
Q

What % of prems have cryptorchidism?

A

30%

How well did you know this?
1
Not at all
2
3
4
5
Perfectly
30
Q

What are the risk factors for cryptorchidism?

A
  1. Advanced maternal age
  2. Maternal Obesity
  3. Low parity
  4. Preterm birth
  5. Low birth weight
How well did you know this?
1
Not at all
2
3
4
5
Perfectly
31
Q

Why is surgical correction needed for undescended testis?

A

Reduced risk of malignancy

How well did you know this?
1
Not at all
2
3
4
5
Perfectly
32
Q

What tests would you order to investigate hematuria?

A
  1. Urinalysis & Micropscopy
  2. Lytes, BUN, Creatinine, Ca, PO,MG

C3/C4 ASOT antiDNA B throat swab

CBC, smear, coags, coombs

HBV/HCV/HIV
IgA

How well did you know this?
1
Not at all
2
3
4
5
Perfectly
33
Q

What are the sequelae of AKI?

A
  1. Fluid overload
  2. Hyperkalemia
  3. HyperPO with hypoCa
  4. ACidosis
  5. Uremia
How well did you know this?
1
Not at all
2
3
4
5
Perfectly
34
Q

What is the definition of nephrotic syndrome range proteinuria?

A

>40 mg/m2/hr

How well did you know this?
1
Not at all
2
3
4
5
Perfectly
35
Q

What is the gold stnadrd for quantification of proteinuria?

A

24 hour urine collection

Second best: protein to creatinine ratio

How well did you know this?
1
Not at all
2
3
4
5
Perfectly
36
Q

How can you distinguish the etiology of proteinuria?

A

Glomerular: large losses of large proteins (albumin, IGG)

TUbular: small losses of small proteins (alpha 1, amino acids)

How well did you know this?
1
Not at all
2
3
4
5
Perfectly
37
Q

What are the characteristics of postural proteinuria?

A
  • Thin, tall, adolescents
  • Less than 1 gram/day
  • “Postural test”: protein free urine during the night, first am increase protein
  • Benign, disppears with age
How well did you know this?
1
Not at all
2
3
4
5
Perfectly
38
Q

What are the characteristics of secondary proteinuria?

A

TRANSIENT under certain conditions: exercise, stress, fever, acute illness, pressors

How well did you know this?
1
Not at all
2
3
4
5
Perfectly
39
Q

What is the work up for proteinuria?

A

24 urine collection

Postural protein test if >10 years

Serum RFTs

Serum albumin and cholesterol

C3/C4

ASOT
Anti DNAse

HCV.HBV.HIV

How well did you know this?
1
Not at all
2
3
4
5
Perfectly
40
Q

What are the compenents of nephrotic syndrome?

A
  • Edema
  • Hypoalbuminemia
  • Albuminuria
  • Hypercholesterolemia
How well did you know this?
1
Not at all
2
3
4
5
Perfectly
41
Q

What is the most common biopsy finding in children with NS?

A

Minimal change nephrotic syndrome

How well did you know this?
1
Not at all
2
3
4
5
Perfectly
42
Q

Who gets MCNS?

A

Males, 2-6, caucasian or asian

How well did you know this?
1
Not at all
2
3
4
5
Perfectly
43
Q

what are the red flags for MCNS (eg. think another diagnosis)?

A
  1. Age <1 year
  2. Age>13years
  3. Black race
  4. Renal failure
  5. Macrohematuria
  6. No response within 4 weeks of steriods
How well did you know this?
1
Not at all
2
3
4
5
Perfectly
44
Q

What are the lab findings in MCNS?

A
  • Proteinuria
  • Decrease in small protein (albumin, OgG, protein C/s)
  • Increase in serum cholesterol

Increase in large proteins (fibrinogen, iGM)

How well did you know this?
1
Not at all
2
3
4
5
Perfectly
45
Q

How can you tell the difference between in underfilled or overfilled nephrotic syndrome?

A

FENA:

<1%: underfilled

How well did you know this?
1
Not at all
2
3
4
5
Perfectly
46
Q

What are the risks of nephrotic syndrome?

A

Infection (SBP)

Thrombosis

Hypercholesterolemia

How well did you know this?
1
Not at all
2
3
4
5
Perfectly
47
Q

What are the characteristics of FSGS?

A

Focal segmental glomerulosclerosis

-More serious, steriod resistant

ESRD if untreated

Steriods

How well did you know this?
1
Not at all
2
3
4
5
Perfectly
48
Q

How do you calculate AG?

A

Na-CL-HCO3

How well did you know this?
1
Not at all
2
3
4
5
Perfectly
49
Q

What are the characteristics of proximal RTA?

A

Acidosis, hypoK, HypoNa, hypoP, gluosuria

How well did you know this?
1
Not at all
2
3
4
5
Perfectly
50
Q

What are 90% of pediatric stones?

A

CALCIUM

How well did you know this?
1
Not at all
2
3
4
5
Perfectly
51
Q

When do the gonads begin to differentiate?

A

6 weeks GA

How well did you know this?
1
Not at all
2
3
4
5
Perfectly
52
Q

What gene causes differentiation?

A

SRY gene

How well did you know this?
1
Not at all
2
3
4
5
Perfectly
53
Q

When does virilization of the external genitalia begin?

A

8 weeks in response to testosterone

How well did you know this?
1
Not at all
2
3
4
5
Perfectly
54
Q

What needs to be functional for testosterone to becomeDHT?

A

5 alpha reductase

Critical for normal male : DHT

How well did you know this?
1
Not at all
2
3
4
5
Perfectly
55
Q

What are the two most common forms of DSD?

A
  1. 46 XX CAH (masculinized female)
  2. 45 X/46 XY Mixed gonadal dysgenesis
How well did you know this?
1
Not at all
2
3
4
5
Perfectly
56
Q

If a pregnancy is conrimed XX CAH, when should dexamethoasone be started?

A

When pregnancy confirmed, <9 weeks

How well did you know this?
1
Not at all
2
3
4
5
Perfectly
57
Q

Are IVP done any more?

A

NO

How well did you know this?
1
Not at all
2
3
4
5
Perfectly
58
Q

What is the role for a VCUG?

A

-Assess for VUR and PUV

How well did you know this?
1
Not at all
2
3
4
5
Perfectly
59
Q

What is the role for a DMSA scan?

A

functional information

Differential scans always =100%

How well did you know this?
1
Not at all
2
3
4
5
Perfectly
60
Q

What are the statistics on congential hydronephrosis?

A

1% of pregnancies

Transient in 44-88%

Not all HN represents significant obstruction

How well did you know this?
1
Not at all
2
3
4
5
Perfectly
61
Q

What is recommended in infants with CH?

A

RBUS

Urgent: bilateral severe CH, solitary kidney

Elective: within 30 days

How well did you know this?
1
Not at all
2
3
4
5
Perfectly
62
Q

What are the four common causes of significant CH (SFU III and IV)?

A
  1. UPJ obstruction
  2. high grade VUR
    UVJ ostruction

PUV

How well did you know this?
1
Not at all
2
3
4
5
Perfectly
63
Q

Which hydronephrosis should be refered to urologY?

A

SFU III and IV

Will need VCUG and diuretic renal scan (4-6 weeks)

How well did you know this?
1
Not at all
2
3
4
5
Perfectly
64
Q

What are the characteristics for posterior urethral valves?

A

Suspect when: male, hydronephrosis, distended thick walled bladder, keyhole sign, oligohydramnios

Urgent postnatal RBUS

Confirmatory VCUG

Surgery

How well did you know this?
1
Not at all
2
3
4
5
Perfectly
65
Q

When is a VCUG indicated?

A
  1. Febrile UTI work up: if AbN RBUS, >1 febrile UTI
  2. Work up of SFU III or IV CH
  3. Work up of late presentation HUN
  4. Follow of VUR if will alter management
  5. Management of neurogenic baldder
  6. Suspicion of PUV
  7. Recurrent cysitis in a prepubertal male (r/o PUV)
How well did you know this?
1
Not at all
2
3
4
5
Perfectly
66
Q

What are the two most common malignant abdominal tumors?

A
  1. Neuroblastoma
  2. Wilms tumor
How well did you know this?
1
Not at all
2
3
4
5
Perfectly
67
Q

What are the recommendations for prevention of renal stones?

A

Increased fluid intake

Avoidance of excess salt intake

Normal calcium intake

How well did you know this?
1
Not at all
2
3
4
5
Perfectly
68
Q

What is the presentation of pyloric stenosis?

A

2 weeks to 2 months

Male x 4 female

Risk factors: FHx+, first born, maternal feeding

How well did you know this?
1
Not at all
2
3
4
5
Perfectly
69
Q

What are the metabolic derangements in pyloric stenosis?

A

Metabolic alkalosis due to loss of electrolytes in vomit (H, CL)

Kidney will try to correct by excreting Na and HCO

When volume contraction occurs, kidney reabsorbs NA and dumps H and K (aldosterone), paradoxical aciuria

How well did you know this?
1
Not at all
2
3
4
5
Perfectly
70
Q

What is the initial fluid replacement in PS?

A

IV bolus of NS until u/o

Maintane fluids with D50.45NS or NS with 20-40 KCL

Aim for Cl>95, HCO<28, K>3.5

How well did you know this?
1
Not at all
2
3
4
5
Perfectly
71
Q

What is the most common type of intussusception?

A

Ileo-colic

How well did you know this?
1
Not at all
2
3
4
5
Perfectly
72
Q

When does intussusception present?

A

1-4/1000

Age 3M-3Y

M 3x F

How well did you know this?
1
Not at all
2
3
4
5
Perfectly
73
Q

What are the absolute contraindications to air enema?

A
  1. Peritonitis
  2. Persistent hypotension
  3. Free air/pneumoperitoneum
How well did you know this?
1
Not at all
2
3
4
5
Perfectly
74
Q

What should you have available when doing an air enema?

A

Angiocath

Abdominal compartment syndrome

How well did you know this?
1
Not at all
2
3
4
5
Perfectly
75
Q

Who does Meckel’s occur in?

A

2% of pop

2 xM to F

2-6% symptomatic

2 years

2 feet from the ileocecal valve

2 inches long

2 types: gastric or pancreatic

How well did you know this?
1
Not at all
2
3
4
5
Perfectly
76
Q

What is the most common presentation of Meckel’s diverticulum?

A

Painless, episodic LGI with drop in HGB

How well did you know this?
1
Not at all
2
3
4
5
Perfectly
77
Q

What are the immediate management guidelines for CDH?

A

Intubate on first breath

NG to decompress

MV< HFO, NO, ECMO

Stable over 24-48 hr then repair

How well did you know this?
1
Not at all
2
3
4
5
Perfectly
78
Q

How does Hirchsprung’s present?

A

Failure to pass meconium within 24 hours

VOmiting, abdo distension

How well did you know this?
1
Not at all
2
3
4
5
Perfectly
79
Q

What is the gold standard for diagnosis of Hirchsprungs?

A

Suction rectal biopsy

How well did you know this?
1
Not at all
2
3
4
5
Perfectly
80
Q

What is the most common malformation associated with omphalocele?

A

TOF

How well did you know this?
1
Not at all
2
3
4
5
Perfectly
81
Q

What is the only indication for contralateral hernia exploration?

A

PREMATURITY

How well did you know this?
1
Not at all
2
3
4
5
Perfectly
82
Q

What is the main indication for orchidoplexy?

A

Risk of infertility

How well did you know this?
1
Not at all
2
3
4
5
Perfectly
83
Q

Which conditions have an increased risk of celiac disease?

A

Down syndrome

Turner syndrome

WIlliam syndrome

Type 1 DM

IgA deficiency

Other autoimmune

FIrst degree relative

(Not russell silver)

How well did you know this?
1
Not at all
2
3
4
5
Perfectly
84
Q

What is celiac disease?

A

Autommune enteropathy with antibodies against gluten?

How well did you know this?
1
Not at all
2
3
4
5
Perfectly
85
Q

What is the gold standrd for diagnosis of celiac?

A

Villious atrophy on intenstinal biopsy

How well did you know this?
1
Not at all
2
3
4
5
Perfectly
86
Q

What contains gluten?

A

WHEAT
RYE
BARLEY

How well did you know this?
1
Not at all
2
3
4
5
Perfectly
87
Q

What is a screening test for celiac?

A

TTG + IGA

How well did you know this?
1
Not at all
2
3
4
5
Perfectly
88
Q

What are the nonGI Sx of celiac?

A

Dermatitis herpetiformis

Dental enamel hypoplasia

Osteopenia

Short stature

Delayed puberty

Iron deficiency anemia

Hepatitis

How well did you know this?
1
Not at all
2
3
4
5
Perfectly
89
Q

What can cause terminal ileitis?

A

Crohns

Lymphoma

Yersinia

TB

CGD

Severe eosinophilic gastroenteropathy

How well did you know this?
1
Not at all
2
3
4
5
Perfectly
90
Q

What are the causes of organic constipation?

A

Hypercalcemia

HypoK

Hypothyroidism

Celiac disease

Lead and mercury poisoning

UC
Medications

CP

NTD

Lactose intolerance

How well did you know this?
1
Not at all
2
3
4
5
Perfectly
91
Q

What are the differences between IgE and non Ige CMPA?

A

IgE: earlier onset (<30 days), FTT, immediate symptoms, FH of atropy

No IgE: later onset (1-4 months), less FTT, delay in symptoms

How well did you know this?
1
Not at all
2
3
4
5
Perfectly
92
Q

Which GI infections must you treat?

A

C diff

Parasite

Consider campylobacter

Support: cholera, yersinia

Don’t: ecoli, shigella, salmonella

How well did you know this?
1
Not at all
2
3
4
5
Perfectly
93
Q

A 3 week old baby has persisting jaundice. Most likely diagnosis?

  1. Gilberts
  2. Biliary atresia
  3. Alpha 1 AT
  4. Inspissated bile syndrome
  5. Physiologic jaundice
A
  1. Gilberts
How well did you know this?
1
Not at all
2
3
4
5
Perfectly
94
Q

What are the characteristics of Gilberts syndrome?

A

7% of the population

AD

Bili >40, <100

50% of infants with persistant unconjugated bili

How well did you know this?
1
Not at all
2
3
4
5
Perfectly
95
Q

What i the most common indication for liver transplant in children?

A

Biliary atresia

How well did you know this?
1
Not at all
2
3
4
5
Perfectly
96
Q

What should you think of if AST>ALT?

A
  1. EtOH
  2. Myopathies
  3. Renal syndromes
  4. Hemolysis
  5. Intestinal inflammation
How well did you know this?
1
Not at all
2
3
4
5
Perfectly
97
Q

What should think of if ALP abnormally low?

A

ZINC

How well did you know this?
1
Not at all
2
3
4
5
Perfectly
98
Q

What are the vitamin K dependent factors?

A

2, 7, 9, 10

How well did you know this?
1
Not at all
2
3
4
5
Perfectly
99
Q

What is the only coagulation factor not made in the liver?

A

Factor VIII

How well did you know this?
1
Not at all
2
3
4
5
Perfectly
100
Q

How do you handle an infant at risk for Hepatitis B?

A

Hep B vaccine within 12 hours

Hep B Ig at the same time, max 7 days

given in the first 24 hours

How well did you know this?
1
Not at all
2
3
4
5
Perfectly
101
Q

What are the causes of chronic pancreatitis?

A

T- toxic, tropical, metabolic

I- idiopathic

G- genetic

A- autoimmune

R- recurrent acute

O- obstructive

How well did you know this?
1
Not at all
2
3
4
5
Perfectly
102
Q

What % of cardiac output goes to the kidney?

A

22%

How well did you know this?
1
Not at all
2
3
4
5
Perfectly
103
Q

What is fanconi syndrome?

A
  1. Tubular proteinuria
  2. Lytes wasting
  3. Glycosuria
How well did you know this?
1
Not at all
2
3
4
5
Perfectly
104
Q

What is cystinosis?

A

AR lysosomal storage d/o

Accumulation of cysteine dimers in lysosomes

Loss of HCO3, PO4, glucose, AA, Na, K

Normal AG metabolic acidosis

How well did you know this?
1
Not at all
2
3
4
5
Perfectly
105
Q

What conditions with hematuria give you low C3?

A

Post infectious GN

MPGN

Lupus nephritis

How well did you know this?
1
Not at all
2
3
4
5
Perfectly
106
Q

What is prune belly syndrome?

A
  1. Abdo muscle deficienct
  2. Severe urinary tract abN
  3. B/L cryptorchidism
How well did you know this?
1
Not at all
2
3
4
5
Perfectly
107
Q

What are the indications for dialysis?

A

1: Acidosis
2. Electrolyte AbN: HyperK, HypoNa, HyperPO4
3. Ingestions (methanol, ethylene glycol, ASA, Li)
4. Fluid overload
5. Uremia

How well did you know this?
1
Not at all
2
3
4
5
Perfectly
108
Q

What are the characteristics of Schwachman-Diamond Syndrome?

A

AR, sBDS mutation

  1. Bone marrow failure
  2. Panreatic dysfunction (second most common to CF)
  3. Skeletal AbN (short stature, osteopenia, dystrophies)
  4. INfections
  5. Myelodysplasia/AML
How well did you know this?
1
Not at all
2
3
4
5
Perfectly
109
Q

How do you diagnose EoE?

A

Biopsy

Supported by CBC, IGE, albumin

How well did you know this?
1
Not at all
2
3
4
5
Perfectly
110
Q

What is the treatment of EoE?

A

Acid suppression

Steriod, PO pulicort for strutures

Dietary elimination (targeted: milk, wheat, soy, eggs, nuts, peanuts, fish, seafood)

How well did you know this?
1
Not at all
2
3
4
5
Perfectly
111
Q

What is selectively absorbed in the distal ileum?

A

Bile salts and Vitamin B12

How well did you know this?
1
Not at all
2
3
4
5
Perfectly
112
Q

How does alpha 1 antitrypsin deficiency present in children?

A

LIVER DISEASE

How well did you know this?
1
Not at all
2
3
4
5
Perfectly
113
Q

How do you investigate for alpha 1 antitrypsin?

A

Serumimmunoassay for alpha 1 antitrypsin

Serum electrophoresis

Genotype by PCR

How well did you know this?
1
Not at all
2
3
4
5
Perfectly
114
Q

What is Behcet disease?

A

Systemic vasculitis HLA B51

Aphthous stomatitis, erthma nodosum and arthritis

How well did you know this?
1
Not at all
2
3
4
5
Perfectly
115
Q

What are the screening tests for celiac disease?

A

IgA TTG

Gliadin

Endomysial Ab

Albumin, CBC, ESR< CRP

How well did you know this?
1
Not at all
2
3
4
5
Perfectly
116
Q

What is the differential diagnosis of bloody stool?

A

IBD

CMPA

NOT CELIAC

How well did you know this?
1
Not at all
2
3
4
5
Perfectly
117
Q

How can you make the diagnosis of pinworms?

A

Direct visualization of the worms in the perianal regoin 2-3 hours after the child goes to sleep

Transparent tape to see eggs in the morning

How well did you know this?
1
Not at all
2
3
4
5
Perfectly
118
Q

WHat is the medical name for hiccups?

A

Intractable singultus

How well did you know this?
1
Not at all
2
3
4
5
Perfectly
119
Q

Coin in esophagus vs. trachea on XR

A

AP: esophagus en face

Lat: trachea en face

How well did you know this?
1
Not at all
2
3
4
5
Perfectly
120
Q

What is the most common cause of epidemic diarrhea worldwide?

A

Norovirus

How well did you know this?
1
Not at all
2
3
4
5
Perfectly
121
Q

When should you measure a stool elastase?

A

Worried abut pancreatic insufficiency

How well did you know this?
1
Not at all
2
3
4
5
Perfectly
122
Q

What are the concentrations of the WHO ORT solution?

A

2% glucose, 20 mEqK/l, 90 mEq Na/L, 80 mEq CL/l, 30 mEq HCO3/l

Add 3/4 tsp salt, 1 tsp bakingsoda, 1 cup orange jioce and 8 tsp sugar to 1 L water

How well did you know this?
1
Not at all
2
3
4
5
Perfectly
123
Q

What is Faget sign?

A

Paradoxical bradycardia with fever in Salmonella (typhoid fever)

How well did you know this?
1
Not at all
2
3
4
5
Perfectly
124
Q

What are the msot common food allergies?

A

Cow milk

Eggs

Peanuts

How well did you know this?
1
Not at all
2
3
4
5
Perfectly
125
Q

What is Sandifer syndrome?

A

Paroxysmal dystonic posturing with opisthotonus and unusual twisting of the head and neck in association with GERD

How well did you know this?
1
Not at all
2
3
4
5
Perfectly
126
Q

How does kwashiorkor and maramus differ?

A

kwashiorkor: edematous malnutrition lowserum oncotic pressure

Marasmus: severe nonedematous malnutrition mixed deficiency of protein and calories.

How well did you know this?
1
Not at all
2
3
4
5
Perfectly
127
Q

When do umbilical hernia warrant Sx?

A

most spontaneously close before 2 y

>1.5 cm at 2Y, ?closure

How well did you know this?
1
Not at all
2
3
4
5
Perfectly
128
Q

When should an inguinal hernia be electively repaired?

A

As soon as possible : incarceration

How well did you know this?
1
Not at all
2
3
4
5
Perfectly
129
Q

What is the classic triad of intussusception?

A
  1. Colicky pain
  2. Vomiting
  3. Passage of bloody stool
How well did you know this?
1
Not at all
2
3
4
5
Perfectly
130
Q

What is the most common type of intussusception?

A

Ileocolic intussusception

How well did you know this?
1
Not at all
2
3
4
5
Perfectly
131
Q

How does biliary atresia present?

A

Recognizable jaundice in the third week of life

Inreasingly dark urine and acholic stools

Spleen enlargement

How well did you know this?
1
Not at all
2
3
4
5
Perfectly
132
Q

What is the classic presentation of pyloric stenosis?

A

3-6 weeks

Progressive non-bilious projectile vomiting

Dehydration with hypoCL, hypoK metabolic alkalosis

How well did you know this?
1
Not at all
2
3
4
5
Perfectly
133
Q

How do you calculate FENa?

A

UNa x Pcreat/ PNa x Ucreat

<1%- Prerenal

>1% Renal

How well did you know this?
1
Not at all
2
3
4
5
Perfectly
134
Q

What are four medications that have changed the outcomes in CKD?

A

EPO= eliminated blood transfusions

Vitamin D= renal osteodystrophy

GH= growth acceleration

ACE/ARB= HTN rx

How well did you know this?
1
Not at all
2
3
4
5
Perfectly
135
Q

GN with low C3?

A

POST infectious

Lupus

Membranoprolierative

C3 glomerulopathy

How well did you know this?
1
Not at all
2
3
4
5
Perfectly
136
Q

What urinary crystals are always pathologic?

A

Cystine

How well did you know this?
1
Not at all
2
3
4
5
Perfectly
137
Q

When should RTA be considered?

A

Chorinc hyperclroemic metabolic acidosis

INability to acidify the ruine

Normal AG

Growth failure, polyuria, polydipsia

How well did you know this?
1
Not at all
2
3
4
5
Perfectly
138
Q

How can you tell apart the RTA?

A
How well did you know this?
1
Not at all
2
3
4
5
Perfectly
139
Q

What is the most common cause of Fanconi syndrome?

A

Cystinosis

How well did you know this?
1
Not at all
2
3
4
5
Perfectly
140
Q

2 year old girl with Ht <5%, blinking, glucose in urine

A

Cystinosis: photophobia, Fanconi syndrome

How well did you know this?
1
Not at all
2
3
4
5
Perfectly
141
Q

For reflux, when would a barium swallow be useful?

A

Useful to rule out mechanical obstruction: malrotation, achalasia

How well did you know this?
1
Not at all
2
3
4
5
Perfectly
142
Q

How long should you try an H2 blocker for reflux before increasing dose, adding or switching therapies?

A

4-6 weeks of same dose

How well did you know this?
1
Not at all
2
3
4
5
Perfectly
143
Q

What are the common infectious causes of pancreatitis (5)?

A
  1. EBV
  2. CMV
  3. Hepatitis A & B
  4. Mycoplasma
  5. Mumps
How well did you know this?
1
Not at all
2
3
4
5
Perfectly
144
Q

What are common medication causes of pancreatitis?
-2 general classes

A

Valproic acid
L-aspariginase
Azathioprine
Mercaptopurine

Think chemotherapy and anti-epileptics

How well did you know this?
1
Not at all
2
3
4
5
Perfectly
145
Q

What are 2 metabolic causes of pancreatitis?

A
  1. Hypercalcemia
  2. Hypertriglyceridemia
How well did you know this?
1
Not at all
2
3
4
5
Perfectly
146
Q

What is the criteria for acute pancreatitis?

A

Need 2/3

  1. Acute RUQ abdominal pain
  2. Elevated amylase or lipase at least 3x normal
  3. Imaging findings consistent with pancreatitis
How well did you know this?
1
Not at all
2
3
4
5
Perfectly
147
Q

What medication should you start when you’re making a patient NPO for a long period of time?

A

Ranitidine or prevacid

How well did you know this?
1
Not at all
2
3
4
5
Perfectly
148
Q

Overview of management of acute pancreatitis?

A
  1. NPO –> can start feeds after 24-48 hrs if improved
    - enteral feeds are preferred to TPN if child is clinically tolerating and not septic
  2. Fluids fluids fluids –> give bolus and 1.5 maintenance
  3. Analgesia –> morphine
  4. Stop any possible aggravating factors
  5. Anti-emetics
How well did you know this?
1
Not at all
2
3
4
5
Perfectly
149
Q

Complications of acute pancreatitis?

A
  1. Pseudocyst
  2. Multi-organ system failure
How well did you know this?
1
Not at all
2
3
4
5
Perfectly
150
Q

Complication of chronic pancreatitis?

A

Diabetes

How well did you know this?
1
Not at all
2
3
4
5
Perfectly
151
Q

What are 3 types of gallstones?

A
  1. Cholesterol stones: increased secretion of cholesterol into bile (see in hyperlipidemia, obesity, pregnancy, females)
  2. Black pigment stones: increased conjugated bilirubin into bile (hemolytic disease, pancreatic insuffiency, TPN)
  3. Brown pigment stones: from bacterial and parasitic infections
How well did you know this?
1
Not at all
2
3
4
5
Perfectly
152
Q

What is the differential diagnosis of hepatitis?

A
  1. Infectious:
    - viral: Hep A/B/C/D/E, adenovirus, coxsackie, enterovirus, EBV, HSV, HIV, VZV, paramyxovirus (RSV, mumps, measles)
    - non-viral: abscess, amebiasis, bacterial sepsis, histoplasmosis, leptospirosis, TB
  2. Autoimmune: autoimmune hepatitis, sclerosing cholangitis, SLE, JIA
  3. Metabolic: alpha-1 antitrypsin deficiency, tyrosinemia, Wilson’s
  4. Toxic: drug-induced (anti-seizure, chemo, acetaminophen)
  5. Anatomic: choledochal cyst, biliary atresia
  6. Hemodynamic: shock, congestive heart failure
  7. Non-alcoholic fatty liver disease: idiopathic, Reye syndrome
How well did you know this?
1
Not at all
2
3
4
5
Perfectly
153
Q

What is the most common presenting complaint of a patient with hepatitis?

A

Jaundice

How well did you know this?
1
Not at all
2
3
4
5
Perfectly
154
Q

What are the 3 main functional liver biochemical profiles in acute liver injury caused by hepatitis viruses?

A
  1. Cytopathic injury
    - rise in serum ALT and AST: magnitude of rise does NOT correlate with extent of hepatocellular necrosis and has little prognostic value
    - slowly improve over several weeks but lag behind serum bilirubin level (normalizes first)
    - rapidly falling ALT and AST in conjunction with increased bilirubin and INR can mean massive hepatic injury (injured or dead cells don’t produce enzymes)
  2. Cholestasis
    - elevated serum conjugated bilirubin from abnormal bile flow at the canalicular and cellular level due to hepatocyte damage and inflammation
    - can also have increased ALP and GGT
  3. Altered synthetic function
    - this should be MAIN FOCUS of monitoring
    - indication for prompt referral to transplant center if abnormal
    - abnormal protein synthesis: decreased coags, decreased albumin
    - metabolic disturbances: hypoglycemia, hyperammonemia, lactic acidosis
    - hepatic encephalopathy: altered LOC with hyperreflexia
How well did you know this?
1
Not at all
2
3
4
5
Perfectly
155
Q

What is Sandifer syndrome?

A

Neck contortions (arching, turning of head) secondary to GERD

How well did you know this?
1
Not at all
2
3
4
5
Perfectly
156
Q

What are the limitations to barium contrast study in GERD investigation?

A

Good for studying the anatomy of the upper GI tract (esophageal strictures, hiatal hernia, gastric outlet or intestinal obstruction) but has poor sensitivity and specificity in the diagnosis of GERD since it has limited duration and the inability to differentiate between physiologic GER from GERD

How well did you know this?
1
Not at all
2
3
4
5
Perfectly
157
Q

What are the indications for a pH probe study in assessing for GERD?

A
  1. Assessing efficacy of acid suppression during treatment
  2. Evaluating apneic episodes in conjunction with a CXR
  3. Evaluating atypical GERD presentations such as chronic cough, stridor and asthma
How well did you know this?
1
Not at all
2
3
4
5
Perfectly
158
Q

What is the treatment approach to GERD?

A
  1. Conservative therapy and lifestyle modification
    -infants: thickening of feeds or use of commercially prethickened formulas (ie. add 1 tbsp of rice cereal per oz of formula), short trial of hypoallergenic diet can be used to exclude milk allergy, positioning measures
    -older children: avoid reflux inducing foods such as tomatoes, chocolate, mint, juice, carbonated and caffeinated drinks, alcohol; weight reduction, stop smoke exposure
  2. Pharmacotherapy
    -mild-moderate reflux esophagitis: histamine-2 receptor
    antagonists
    -severe reflux esophagitis: PPIs (more potent than H2 blockers but more expensive)
    -2nd line: prokinetic agents (metoclopramide, erythromycin) = increase LES pressure or improve gastric emptying
How well did you know this?
1
Not at all
2
3
4
5
Perfectly
159
Q

What is the black box warning for metoclopramide?

A

Chronic use (>3 mo) linked with tardive dyskinesia

How well did you know this?
1
Not at all
2
3
4
5
Perfectly
160
Q

What condition must always be ruled out in a neonate presenting with bilious emesis?

  • diagnostic test?
  • Where is the obstruction in a patient with bilious emesis?
A

Malrotation with volvulus

  • UGI series with small bowel follow through and STAT gen surg consult
  • obstruction: distal to the ampulla of vater (halfway along the duodenum, formed by the pancreatic duct and common bile duct)
How well did you know this?
1
Not at all
2
3
4
5
Perfectly
161
Q

Which test has high sensitivity and positive predictive value for bacterial gastroenteritis?

A

Stool fecal leukocytes

How well did you know this?
1
Not at all
2
3
4
5
Perfectly
162
Q

What is the most common cause of diarrhea resulting in hospitalization in children?

A

Rotavirus

How well did you know this?
1
Not at all
2
3
4
5
Perfectly
163
Q

What are the infectious causes of bloody (ie. inflammatory) diarrhea? (7)

A

Inflammatory diarrhea: organisms and cytotoxins invade mucosa, killing mucosal cells = bleeding

  • think EECYSTS
    1. E coli (0157:H7)
    2. E. histolytica
    3. Campylobacter/C. diff
    4. Yersinia
    5. Shigella
    6. Salmonella Typhi
    7. Strongyloides
How well did you know this?
1
Not at all
2
3
4
5
Perfectly
164
Q

What is the purpose of always treating salmonella bacteremia in infants

A

Salmonella bacteremia can quickly become invasive and cause sepsis, meningitis, especially in infants

How well did you know this?
1
Not at all
2
3
4
5
Perfectly
165
Q

What is the antibiotic treatment for severe shigella acute diarrhea?

A

Azithromycin
-if it is NOT severe, do NOT treat as this may worsen chance of HUS (endotoxin release)

How well did you know this?
1
Not at all
2
3
4
5
Perfectly
166
Q

What is the antibiotic treatment for severe yersinia acute diarrhea?

A

Ceftriaxone

How well did you know this?
1
Not at all
2
3
4
5
Perfectly
167
Q

A patient presents with acute vomiting and diarrhea after eating reheated rice. What organism is most likely the culprit?
-management?

A

Bacillus cereus
-management: supportive care

How well did you know this?
1
Not at all
2
3
4
5
Perfectly
168
Q

A patient presents with acute diarrhea after eating raw shell fish. What organism is most likely the culprit?
-treatment?

A

Vibrio cholerae
-supportive care OR doxycycline for severe illness

How well did you know this?
1
Not at all
2
3
4
5
Perfectly
169
Q

A patient presents with acute diarrhea after playing in the dirt. He is also found to have peripheral eosinophilia and a liver abscess. What organism is most likely the culprit?
-treatment?

A

Entamoeba histolytica
-mebendazole

How well did you know this?
1
Not at all
2
3
4
5
Perfectly
170
Q

In a patient with chronic constipation, what 3 signs are suggestive of a distal GI obstruction and should prompt further investigations?

A
  1. Narrow diameter stools (stools squeezing past an obstruction or Hirschsprung’s where not enough strength is generated to push the entire stool mass through)
  2. Abdominal distention
  3. Lack of encopresis (almost never see encopresis in Hirschsprung’s)
How well did you know this?
1
Not at all
2
3
4
5
Perfectly
171
Q

What are the 3 most common causes of chronic/recurrent abdominal pain in children?

A

In order of prevalence:

  1. Functional abdominal pain
  2. Lactose intolerance
  3. Constipation
How well did you know this?
1
Not at all
2
3
4
5
Perfectly
172
Q

How do you differentiate upper vs. lower GI bleed anatomically?

A

Upper = above ligament of Treitz
Lower = below ligament of Treitz
(suspensory ligament attaching duodenum to connective tissue)

How well did you know this?
1
Not at all
2
3
4
5
Perfectly
173
Q

What is the differential diagnosis for hematemesis +/- melena? (7)

A
  1. Swallowed blood (epistaxis, dental work, etc.)
  2. esophagitis
  3. gastritis
  4. peptic ulcer disease
  5. mallory-weiss tear
  6. esophageal varices
  7. vascular malformations
How well did you know this?
1
Not at all
2
3
4
5
Perfectly
174
Q

What is the differential diagnosis for hematochezia? (9)

A
  1. CMPA
  2. NEC
  3. Meckel diverticulum
  4. Vasculitis (HSP)
  5. Polyp
  6. Intestinal or colonic ulcer
  7. Colitis (infectious vs. IBD)
  8. Vascular malformation
  9. Anal fissure
How well did you know this?
1
Not at all
2
3
4
5
Perfectly
175
Q

What are the 2 most common causes of bloody stools in infants

A
  1. CMPA
  2. Anal/rectal fissure
How well did you know this?
1
Not at all
2
3
4
5
Perfectly
176
Q

What is a common cause of perianal dermatitis in toddlers and school-age children?

  • clinical features?
  • diagnosis?
  • -work-up to rule out other causes?
  • treatment?
A

GAS!

  • clinical features: perianal erythema, pain, pruritis
  • diagnosis: rapid strep test or culture
  • workup: CBC, CRP, ESR, gonorrhea/chlamydia swabs, tape test for pinworms
  • treatment: 10 d course penicillin
How well did you know this?
1
Not at all
2
3
4
5
Perfectly
177
Q

What is the initial imaging study of choice for a patient presenting with hepatosplenomegaly?

A

Abdominal U/S

How well did you know this?
1
Not at all
2
3
4
5
Perfectly
178
Q

What imaging study can help differentiate between biliary atresia and neonatal hepatitis in a neonate presenting with conjugated hyperbilirubinemia?

A

HIDA scan (radionuclide scan)

  • in biliary atresia: hepatic uptake of radionuclide is normal but there is no excretion into the intestines
  • in neonatal hepatitis: hepatic uptake of radionuclide is normal but excretion into the intestines is normal
How well did you know this?
1
Not at all
2
3
4
5
Perfectly
179
Q

What is the most common cause of indirect hyperbilirubinemia in a patient > 1 month of age?

A

Gilbert disease = benign disorder of mildly impaired bilirubin conjugation found in 5-10% of population

180
Q

What is the differential diagnosis for conjugated hyperbilirubinemia?

  • intrahepatic (9)
  • extrahepatic (4)
A

Results from direct obstruction of bile flow from the liver into the biliary system, then into the intestine (can be from obstruction inside the liver or outside the liver) = aka cholestasis

Intrahepatic:

  1. Alagille syndrome (congenital abnormalities of the intrahepatic biliary tract)
  2. TPN
  3. Wilson disease
  4. Viral hepatitis
  5. Autoimmune hepatitis
  6. Metabolic disorders (ie. Dubin-Johnson syndrome)
  7. Drug-induced cholestasis
  8. Alpha-1 antitrypsin deficiency
  9. Neonatal hepatitis

Extrahepatic:

  1. Biliary atresia
  2. Choledochal cyst
  3. Abdominal mass
  4. Obstructive gallstone
181
Q

What is the differential diagnosis for unconjugated hyperbilirubinemia? (8)

A

Results from increased RBC turnover or impaired conjugation

  1. Breastmilk jaundice
  2. Breastfeeding jaundice (neonatal physiologic jaundice)
  3. Gilbert disease
  4. Hemolytic anemia
  5. Crigler-Najjar syndrome
  6. Hypothyroidism
  7. Cystic fibrosis
  8. Viral hepatitis
182
Q

What imaging should always be ordered for all cases of direct hyperbilirubinemia and should always be your initial imaging study of choice?

A

Abdominal U/S

183
Q

What is achalasia?

  • gold standard for diagnosis?
  • classic sign on UGI?
A

Loss of LES relaxation due to high resting LES pressure and absent or non-peristaltic esophageal contractions

  • gold standard for diagnosis: esophageal manometry)
  • UGI = “bird’s beak” = tapering of esophagus
184
Q

What are 3 possible complications from untreated GERD?

A
  1. Esophageal strictures
  2. Barrett’s esophagus
  3. Adenocarcinoma
185
Q

How do yuo diagnose eosinophilic esophagitis?

A
  1. Bloodwork: peripheral eosinophilia, elevated IgE
  2. Endoscopy = gold standard = biopsy showing >15-20 eosinophils/hpf
186
Q

An immunocompromised patient with HIV presents to you with fever, dysphagia, odynophagia, and retrosternal pain. What is the most likely diagnosis?

A

Infectious esophagitis
-common etiologies: candida, HSV, CMV

187
Q

Which medication classes are the most common causes of pill esophagitis? (3)

  • treatment?
  • prevention?
A

Pill esophagitis: irritation/erosion of the esophageal wall due to a pill temporarily lodging in the esophagus

  1. Antibiotics - specifically tetracycline, doxycycline, cilndamycin
  2. Anti-inflammatories - aspirin
  3. Bisphosphanates
    - treatment: stop the offending drug if possible; if not, switch to liquid formulation
    - prevention: always take pills standing upright, with at least 8 oz of water
188
Q

What are the clinical features of esophageal perforation?

  • diagnosis?
  • treatment?
A
  1. Acute onset of severe epigastric/retrosternal pain and dysphagia
  2. Looks unwell with fever, tachycardia, abdominal pain
    - diagnose by obtaining C spine and CXR = will see free retrosternal air and pneumomediatstinum
    - if xrays are normal but you have a high suspicion, get an esophagram with water-soluble contrast
    - treatment: NPO, gastric drainage, broad spectrum IV abx for small esophageal tears in stable patients. If unstable or large esophageal teams, need emergent surgical repair
189
Q

What is the treatment for ruptured esophageal varices?
-pharmacological vs. surgical

A

Pharmacological: IV octreotide
surgical: endoscopic sclerotherapy, band ligation or thrombin injection

190
Q

What are the usual causes of and location of ulcers in:

  • primary peptic ulcer disease (1)
  • secondary peptic ulcer disease (2)
  • how to differentiate between gastric and duodenal ulcers clinically?
A

Primary PUD: usually caused by H. pylori, duodenal location
Secondary PUD: gastric in location, usually caused by:
1. hypergastrinemia
2. Zolllinger-Ellison syndrome: autonomous gastrin secretion by gastrinoma, associated with MEN-I (recurrent, multiple, atypically located ulcers and treated with PPI and octreotide to inhibit tumor growth)
***gastric ulcers = pain worsens with meals, especially with spicy foods
***duodenal ulcers = pain improves with meals

191
Q

What are risk factors for developing peptic ulcer disease?

  • test for diagnosis?
  • treatment?
A
  1. Caffeine/alcohol/tobacco use
  2. G tubes
  3. NSAIDs
  4. Burn injuries
  5. Systemic illnesses: sepsis
  6. Steroids
    -gold standard for diagnosis: endoscopy with biopsy
    -treatment:
  7. Avoid triggering substances
  8. PPIs
  9. Treatment of H pylori with clarithromycin +amoxil (abx x 2 wks) + PPI (x 1 mo)
    (OR amox + flagyl + PPI OR clarithro + flagyl + PPI)
  10. Surgery for severe and/or refractory cases
192
Q

What is malrotation and why does it increase risk for volvulus?

A

Developmental abnormality in which there is incomplete rotation of the midgut and thus, the cecum is in the RUQ instead of the RLQ and the mesentary is fixed on a narrow base
-this places the small bowel at increased risk for twisting on the pedicle of mesentary

193
Q

In what age group is volvulus most likely seen in?
-what is the definitive treatment?

A

Early infancy = more than 90% of cases present in first year of life BUT can present at any age
-definitive treatment: surgery (Ladd procedure)

194
Q

What is the rule of 2s for meckel diverticulum? (6)

A
  1. 2:1 M:F ratio
  2. 2% of population
  3. 2 feet above ileocecal valve
  4. 2 year of age most common presentation
  5. 2 inches in size
  6. 2 types of tissue: gastric and pancreatic
195
Q

What is Meckel diverticulum?

  • clinical features?
  • diagnostic test?
  • treatment?
A

Meckel diverticulum: remnant of embryonic yolk sac that results from failure of the omphalomesenteric/vitteline duct to involute.

  • clinical features:
    1. PAINLESS rectal bleeding (gastric mucosa secrets acid leading to ulceration and bleeding of intestinal mucosa)
    2. Can also present with intussusception or volvulus (acts as lead point)
  • diagnostic test: Meckel scan (technietium-99 radionuclide scan)
  • treatment: surgical excision
196
Q

What is the most common lead point in an older child with intussusception?

A

Meckel diverticulum

197
Q

You have just completed an air enema with successful reduction of an intussusception. What do you do now?

a. admit for observation
b. d/c home
- what is the reduction rate with air enema?
- what is the recurrence rate?

A

Admit for observation x 24-48 hrs = recurrence can occur

  • reduction rate with air enema: 80-95%
  • recurrence rate after reduction: 10%
198
Q

A patient presents to you with severe, progressive epigastric pain with intractable vomiting. 3 days ago, he sustained a bicycle handlebar injury to the abdomen. What is the most likely diagnosis?

  • imaging?
  • treatment?
A

Duodenal hematoma = swelling causes intestinal obstruction

  • diagnosis: UGI series (“coiled spring appearance”) or CT abdo
  • treatment: NPO, NG decompression, TPN until obstructive symptoms resolve (usually 7-10 d)
199
Q

What is SMA syndrome?

  • main risk factors (2)?
  • clinical features?
A

SMA syndrome = compression of the duodenum by the SMA against the aorta

  • 2 main risk factors:
    1. rapid weight loss from malnutrition or catabolic states = loss of mesenteric fat pad which collapses the duodenum in between the SMA and the aorta
    2. full body cast (extra-abdominal compression)
  • clinical features:
    1. Severe epigastric pain
    2. Vomiting
200
Q

An adolescent who has been in a body cast following orthopedic surgery starts vomiting profusely and complains of severe epigastric pain 2 weeks post-op. What is the most likely diagnosis?
-treatment?

A

SMA syndrome

  • treatment:
    1. Lateral or prone positioning to shift duodenum away from obstructing structures and resme oral intake
    2. If that doesn’t work, may need NJ tube or TPN
201
Q

What are the differences between Crohn’s disease and ulcerative colitis on gross pathology?
-on histology?

A

Gross pathology:

  • CD: cobble stoning, strictures, fistulas, fissures
  • UC: diffuse continuous inflammation

Histology:

  • CD: transmural involvement
  • UC: mucosal and submucosal inflammation, crypt abscesses
202
Q

What is perianal disease (tags, fissures, abscesses) more commonly seen in: CD or UC?

A

CD

203
Q

What are the possible extraintestinal manifestations seen in Crohn’s disease? (6)

A
  1. Spondyloarthropathy
  2. Pyoderma gangrenosum (painless, necrotic tissue)
  3. Erythema nodosum (PAINFUL)
  4. Aphthous ulcers
  5. Uveitis
  6. Primary sclerosing cholangitis
204
Q

What three extraintestinal manifestations are more commonly seen in UC than CD?

A
  1. Pyoderma gangrenosum
  2. Primary sclerosing cholangitis
  3. Thrombosis
205
Q

Which do you see the following more commonly in, UC or CD?

  • toxic megacolon
  • risk for cancer
  • pANCA positive
A

All 3 are more common in UC

206
Q

What is the treatment for IBD, in order of increasing disease severity? (7)

A
  1. Steroids
  2. Mesalamine
  3. Sulfasalazine
  4. azathioprine
  5. Methotrexate
  6. Infliximab
  7. Surgical resection
207
Q

What are associated conditions with celiac disease? (5)

A
  1. Autoimmune thyroiditis
  2. Type 1 DM
  3. Down syndrome
  4. Turner syndrome
  5. IgA deficiency
  6. William’s syndrome
208
Q

What are 8 intestinal manifestations of celiac disease?

  • what are 5 signs on physical exam of celiac disease?
  • what are 3 biopsy findings of celiac disease?
  • what are non GI manifestations of celiac disease?
A

Intestinal manifestations:

  1. FTT/wt loss
  2. Diarrhea (with occult blood loss)
  3. Irritability
  4. Vomiting
  5. Anorexia or excessive appetite
  6. Foul-smelling, bulky stools
  7. Abdominal pain
  8. Rectal prolapse

Signs:

  1. Poor growth for both height and weight
  2. Wasted muscles
  3. Abdominal distention
  4. Edema
  5. Digital clubbing

Biopsy findings:

  1. Villous atrophy
  2. Crypt hyperplasia
  3. Intraepithelial lymphocytosis

Non GI manifestations:

  1. Dermatitis herptiformis
  2. Dental enamel hypoplasia of permanent teeth
  3. Iron deficiency anemia resistant to oral Fe
209
Q

What is the initial testing for celiac disease?
-definitive diagnosis?

A

Initial testing: elevated tissue transglutaminase (can be falsely negative in IgA deficiency so ALWAYS order an IgA with tissue transglutaminase)
-definitive diagnosis: small intestinal biopsy showing villous atrophy

210
Q

A mom of a 2 yo girl tells you she thinks her daughter is lactose-intolerant. What is your response?

A

IDIOT. Acquired lactase deficiency is VERY rare in children

211
Q

What is the most common syndrome associated with TEF?

A

VACTERL

  • Vertebral anomalies
  • Anal atresia
  • Cardiac defects
  • TracheoEsophageal fistula
  • Renal abnormalities
  • Limb abnormaliies
212
Q

What is the perforation rate for appendicitis in infants compared to the general population?
-reason for the difference?

A

Perf rate in infants: 50%
Perf rate in general population: 10%
-due to delayed diagnosis/atypical presentation

213
Q

What is the “hamburger sign” in appendicitis?

A

Refers to the fact that patients with appendicitis will usually not even want to eat their favourite food due to nausea/anorexia. Thus, if they’re hungry, it’s probably NOT appendicitis

214
Q

How do you perform:

  • rovsing’s sign?
  • psoas sign?
  • obturator sign?
  • sensitivity of these tests?
A

Rovsing’s sign: press in the LLQ, if the patient feels pain in RRQ, this is positive

Psoas sign = seen with retrocecal appendicitis
-so remember that the psoas muscle is posterior SO, get the patient to roll onto their left side and EXTEND the right hip. If this causes the patient pain, this is a positive psoas sign

Obturator sign: obturator muscle causes internal rotation of the hip SO flex the patient’s hip and knee and internally rotate. If painful, this is a positive obturator sign

****These tests have high specificity but LOW sensitivity

215
Q

What is the treatment of a perforated appendicitis?

A

IV antibiotics = amp/gent/flagyl until symptomatically improved +/- percutaneous drainage of abscess if present

216
Q

What is the most common cause of acute liver failure in children?

A

Viral hepatitis

217
Q

What are the 2 forms of autoimmune hepatitis?
-target population?

A

Type I autoimmune hepatitis: usually in young women aged 15-25 yrs, associated with other autoimmune conditions
-type II autoimmune hepatitis: occurs in young children

218
Q

What labwork confirms a diagnosis of autoimmune hepatitis? (3)

  • definitive diagnosis?
  • treatment?
A
  1. Hypergammaglobulinemia
  2. Positive ANA and anti-smooth-muscle antibodies (70% positive on presentation); some patients have anti-liver-kidney antibodies
  3. Elevated transaminases
    - definitive diagnosis: liver biopsy
    - treatment: steroids +/- azathioprine, may need liver transplant if ESLD
219
Q

What are the clinical manifestations of Wilson’s disease?

  • diagnosis?
  • treatment?
A
220
Q

A patient presents to you 1 week after a viral illness with vomiting, irritability and lethargy. She then develops delirium and seizures. There is mild hepatomegaly and her bloodwork reveals elevated liver enzymes, ammonia, coagulopathy and hypoglycemia. Liver biopsy shows elevated triglyceride content and diffuse fatty infiltration of hepatocytes with minimal inflammatory changes. What is your diagnosis?

A

Reye syndrome! Acute encephalopathy and fatty degeneration of the liver

  • associated with aspirin use in children with viral infections including influenza and varicella
  • think encephalopathy, liver failure, and FATTY LIVER on biopsy
221
Q

What is NASH?

  • 2 risk factors?
  • treatment?
A

Non-alcoholic steatohepatitis

  • fatty infiltration of the liver
  • 2 risk factors:
    1. obesity
    2. type 2 DM
  • treatment: weight loss through diet and exercise
222
Q

What is the pathophysiology of hereditary tyrosinemia?

  • clinical features?
  • what do you see on labwork?
  • treatment?
A

AR condition = deficient fumarylacetoacetate hydrolase = affects liver, kidney, peripheral nervous system
-clinical features (begin in infancy):
1. FTT
2. Jaundice
3. HSM with increased risk of hepatocellular carcinoma
-labwork:
1. Elevated tyrosine
2. Elevated AFP
3. Elevated urine
succinylacetone
-treatment: dietary restriction of tyrosine, phenylalanine, methionine, medical management with tyrosine degradation inhibitor, supportive care for liver failure

223
Q

What is the management principles of portal hypertension? (4)

A
  1. Treament of underlying disease
  2. Medications to decrease portal pressure (beta blockers, vasopressin)
  3. Surgical portosystemic shunt +/- liver transplant
  4. Treat complications (ie. esophageal varices)
224
Q

What are the 2 most common organisms causing spontaneous bacterial peritonitis?
-empiric treatment?

A
  1. E. coli
  2. Klebsiella
    - empiric treatment: IV ceftriaxone
225
Q

What is the clinical criteria for diagnosis of fulminant hepatic failure?
-what percentage of cases are idiopathic?

A

Need 3/3:

  1. Biochemical evidence of acute liver injury
  2. No evidence of chronic liver disease
  3. Hepatic-based coagulopathy defined as:
    a. In presence of hepatic encephalopathy: PT > 15 sec or INR > 1.5 not corrected by vitamin K
    b. In absence of hepatic encephalopathy: PT > 20 sec or INR > 2

-50% of cases are idiopathic

226
Q

What are the management principles for fulminant hepatic failure? (6)

A
  1. Neurologic: avoid sedatives, limit protein intake (to decrease ammonia), neurovitals and monitor for increased ICP due to cerebral edema
  2. Respiratory: intubation to prevent aspiration due to impaired LOC
  3. GI: antacids, glucose control
  4. Renal: avoid hypovolemia (risks hepatorenal syndrome)
  5. Hematology: give vitamin K, FFP, platelets, etc
  6. Liver transplant (mortality is 70% without transplant)
227
Q

What is the most common condition requiring liver transplant in the pediatric population?

  • clinical presentation?
  • diagnosis?
  • treatment?
A

Biliary atresia: atresia of any portion of the extrahepatic biliary system = 50% of liver transplants are because of biliary atresia!

  • clinical presentation: conjugated hyperbilirubinemia in an otherwise healthy, growing infant in the first few months of life
  • diagnosis: elevated liver enzymes, conjugated hyperbilirubinemia, HIDA scan showing lack of excretion of tracer from liver into the intestinal tract
  • definitive diagnosis: liver biopsy showing bile duct proliferation and widened portal area
  • treatment: Kasai procedure, eventual liver transplant
228
Q

What is Charcot’s triad?

  • What is Reynold’s pentad?
  • seen in what condition?
A

Charcot’s triad:

  1. Fever
  2. RUQ pain
  3. Jaundice

Reynold’s pentad:

  1. Fever
  2. RUQ pain
  3. Jaundice
  4. Hypotension
  5. Altered mental status

****See both in ascending cholangitis

229
Q

What are the clinical features of Alagille syndrome? (6)

A
  1. Ocular abnormalities: posterior embryotoxin
  2. Vertebral anomalies: butterfly vertebrae
  3. Intrahepatic bile duct paucity
  4. Triangular face
  5. Cardiac defects: peripheral pulmonic stenosis, pulmonary valve stenosis, etc.
  6. Renal abnormalities
230
Q

How many types of choledochal cysts are there?

  • which is most common type?
  • which population does it affect more: males or females?
A

5 types overall

  • Type I = most common = diltation of the common bile duct
  • affects females 4x more than males
  • type 5 = Caroli disease = intrahepatic bile duct cysts
231
Q

A patient presents with focal biliary cirrhosis. What condition is this pathognomonic for and what testing should you do?

A

Pathognomonic for CF! Need to do sweat chloride

232
Q

What is ascending cholangitis?
-treatment?

A

Bacterial infection of obstructed common bile duct usually due to stricture of stones

  • see Charcot’s triad or Reynold’s pentad
  • treatment: after diagnosis via ultrasound/ERCP/percutaneous transhepatic cholangiography, do NPO, antibiotics, stone removal or dilation via ERCP or PTC
233
Q

What is primary sclerosing cholangitis?

  • labwork?
  • imaging?
  • treatment:
A

Inflammation and fibrosis of hepatobiliary system (occurs in

234
Q

What are the similarities (4) and differences (1) between Familial Adenomatous Polyposis and Gardner syndrome?

A

Similarities:

  1. Both are AD
  2. Both have average onset in 2nd decade
  3. Both present with rectal bleeding, abdo pain, bowel obstruction
  4. Both have 100% risk of colon cancer

Difference:
1. FAP has adenomas in large intestine only while Gardner syndrome has adenomas in both small and large intestine

235
Q

What feeds are appropriate for short-gut syndrome?
-how to start feeds?

A

Protein-hydrolyzed formula with MCT in order to ensure absorption (ie. alimentum with medium chain triglycerides)
-start with TPN, then small enteral feeds, then slowly increase volume in order to increase pancreatic-biliary flow

236
Q

What are 4 possible lab findings of CMPA?

A
  1. Increased IgE
  2. Increased eosinophils
  3. Increased platelets
  4. Decreased albumin
237
Q

What is the most common cause of protein losing enteropathy?

  • clinical features?
  • treatment?
A

CMPA

  • clinical features: edema, hypoalbuminemia, iron deficiency anemia, vomiting, bloody stools, irritability
  • Treatment: quick response to removal of cow’s milk protein (remember that 50% react also to soy protein)
  • need hydrolyzed protein formula (nutramigen, progestamil, alimentum) OR if this fails, then use amino-acid based formula (neocate)
  • most grow out of CMPA and tolerate cow’s milk by 2 yr
238
Q

What test should you order if you want to diagnose a carbohydrate malabsorption condition?

A

Stool reducing substances

239
Q

How long does breastmilk jaundice last for?

  • peak?
  • how to confirm?
A

Can last up to 10 weeks

  • peaks at 5-15 days
  • confirm with trial of pumping
240
Q

How common is lactose intolerance in asians vs. blacks?
-is testing required?

A

Asians: 40%
Blacks: 85%
-testing is not required if symptoms improve with removal of dairy
-hydrogen breath test if you need to confirm

241
Q

What are extrahepatic causes of portal hypertension? (3)

A
  1. Portal vein thrombosis (most common)
  2. Portal vein agenesis/stenosis
  3. Splenic vein thrombosis
242
Q

What is Peutz-Jehgers?

  • clinical features
  • potential complications? (3)
A

Autsomal dominant disease causing hamartomas
1. Hyperpigmented lesions on lips and oral mucosa (other mucosal areas can be involved too)
2. Hamartomas in small and large bowel (also elsewhere in GI)
Complications:
1. hamartomas can serve as lead point for intussusception
2. GI malignancy: yearly colonoscopy needed
3. Breast/testicular malignancy

243
Q

How do you diagnose H. pylori?

A
  1. Hydrogen breath test
  2. Biopsy
244
Q

What are the 2 most common complications of gastroschesis repair?

A
  1. Abdominal compartment syndrome
  2. NEC
245
Q

What is the definitive diagnostic test for peptic ulcer disease?
-supportive test?

A

Endoscopy for biopsies and H pylori cultures
-supportive test: Hydrogen breath test

246
Q

What are 2 lab findings in eosinophilic esophagatisi?
-appearance on endoscopy?

A
  1. Peripheral eosinophilia
  2. Elevated IgE
    - appearance on endoscopy: FURROWED esophagus
247
Q

What is the treatment for eosinophilic esophagitis?

A
  1. Dietary restriction of major food allergens (usually milk, soy, wheat, egg, peanuts and treenuts, seafood)
  2. Topical and systemic corticosteroids
248
Q

What are signs of meconium ileus on abdominal imaging (ie. AXR/contrast enema)?
-most common area of obstruction?

A

AXR: dilated bowel loops at level of obstructon (usually at terminal ileum)

  • see bubbly, granular pattern at the level of obstruction
  • Contrast enema: microcolon from disuse
249
Q

How can you differentiate on clinical history GERD vs. EE?

A

EE usually does NOT respond to acid blockade therapy

250
Q

What is the most serious complication of ulverative colitis?

A

Toxic megacolon

251
Q

What is the pathophysiology underlying Hirschsprung disease?

A

Absence of ganglion cells in the submucosal and myenteric plexus = inadequate relaxation of the bowel wall with bowel wall hypertonicity

252
Q

What is the most common of lower intestinal obstruction in neonates?
-male:female ratio?

A

Hirshsprung/s disease
-M:F = 4:1 for short segment disease

253
Q

What findings on physical examination can help differentiate between constipation vs. Hirschsprung’s? (3)
-on history? (2)

A

Hirschsprung’s:

  1. Abdominal distension
  2. Empty rectum on DRE with possible explosive passage of stool
  3. Possible poor weight gain
    - on history:
  4. Onset of constipation at birth (for functional constipation, usually after 2 yr of age)
  5. Encopresis VERY rare
254
Q

What are tests to order for diagnosing Hirschsprung’s? (3)
-which one is gold standard?

A
  1. Anorectal manometry: initial diagnostic test = will see failure of internal anal sphincter relaxation in response to dilating a balloon in the rectum
  2. Barium enema: delayed evacuation > 24 hrs
  3. Rectal biopsy = GOLD STANDARD = will see no ganglion cells, increased acetylcholinesterase staining
255
Q

What are clinical features of Giardia infection? (5)

A
  1. Recurrent abdmoinal pain
  2. Cramping
  3. Bloating
  4. Weight loss
  5. Intermittent diarrhea
256
Q

What is the order of teeth eruption in children?

A
  1. Mandibular central incisors: 5-7 mo
  2. Maxillary central incisors: 6-8 mo
  3. Lateral incisors (mandibular first, then maxillary): 7-11 mo
  4. First molars: 10-16 mo
  5. Cuspids: 16-20 mo
  6. 2nd molars: 20-30 mo
257
Q

What is the charactsteristic pattern of tooth decay seen with prolonged use of a baby bottle?
-when should weaning from bottle occur?

A

Extensive maxillary decay (especially frontal) and posterior maxillary and manibular delay but NORMAL mandibular frontal teeth
-weaning from bottle: when they turn 1 yo

258
Q

What is the rate of success of replantation of permanent teeth at:

  • 30 minutes
  • >2 hrs
  • what to do if a permanent tooth is knocked out?
  • management of loss of primary tooth?
A
  • @ 30 minutes: 90% chance
  • @ > 2 hrs: t need to replace since it may damage the underlying permanent tooth if shoved back in
259
Q

In a patient with suspected E. coli 0157:H, what test can isolate the organism?

A

Stool culture on sorbitol-MacConkey medium (won’t grow on regular stool cultures)

260
Q

A 17 mo old toddler presents with irritability, refusal to walk with tenderness in both of her legs. She has a low grade fever, petechiae on her skin and mucous membranes. She has a small cut that has not healed well. Radiographs of the legs show bony atrophy with epiphyseal separation. What is the most likely diagnosis?

A

Vitamin C deficiency (ie. scurvy)

261
Q

What are common acute complications of Crohn’s disease?

A
  1. Abscesses.
  2. Perforation
  3. Toxic megacolon (less risk than UC)
  4. GI bleed
  5. Bowel obstruction
  6. Infection
262
Q

What are contraindications to air or barium enema in treating intussusception? (3)

A
  1. Perforation
  2. Hemodynamically unstable
  3. Signs of ischemia
263
Q

What are circumstances that air enema would not be effective in reducing intussusception? (2)

A
  1. Pathologic lead point
  2. Ileo-ileo intussusception (usually doesn’t respond to air enema, usually requires surgery)
264
Q

What are risk factors for adenocarcinoma in the setting of a diagnosis of UC? (4)

A
  1. Disease > 10 years
  2. Onset before age 15 yo
  3. Pancolitis
  4. PSC
265
Q

What are life threatening complications for UC? (5)

A
  1. Sepsis
  2. Primary sclerosing cholangitis
  3. Fulminant colitis
  4. Toxic megacolon/perforation
  5. Adenocarcinoma
266
Q

What is the the differential diagnosis for terminal ileitis? (5)

A
  1. Crohn’s disease
  2. Yersinia infection
  3. TB
  4. Lymphoma
  5. CGD
267
Q

What are organic causes of constipation? (6)

A
  1. Hypercalcemia
  2. Hypokalemia
  3. Hypothyroidism
  4. Celiac disease
  5. Hirschsprung’s
  6. Ulcerative colitis
268
Q

What is the diagnostic criteria for irritable bowel syndrome?

A
  1. More than 1 week of symptoms for 2 or more months
  2. Abdominal discomfort or pain associated with at least 2 of the following:
    a. Improved with defecation
    b. onset associated with change in frequency of stool
    c. onset associated with change in form of stool
  3. No organic cause
269
Q

What is your ddx if AST >> ALT? (5)
-what is the most likely cause of an ALP is abnormally low?

A
  1. ETOH
  2. Myopathy
  3. Renal syndromes
  4. Hemolyasis
  5. Adenovirus infection

****Abnormally low ALP: think zinc deficiency

270
Q

What are causes of small-intestinal flat villi? (8)

A
  1. Giardia
  2. HIV
  3. Intestinal TB
  4. Primary immunodeficiency
  5. Crohn’s disease
  6. Celiac’s
  7. Malnutrition
271
Q

What are risk factors for TPN cholestasis? (5)

A
  1. Prematurity
  2. Low birth weight
  3. Sepsis
  4. Prolonged duration of TPN
  5. NEC
272
Q

Fill in the blank: Problems with filtration are due to problems with _________ whereas problems with secretion and reabsorption are due to problems with _______.

  • what is the definition of GFR?
  • what is used as an estimate for GFR?
A

Problems with filtration: glomeruli = act as sieves to filter serum, creating ultrafiltrate
Problems with secretion and reabsorption: tubules = reabsorb and secrete solutes, concentrate the urine and create and respond to endocrine signals
-GFR = glomerular filtration rate = amount of ultrafiltrate produced by all of the glomeruli (ml/min). Varies with age and height
-Creatinine clearance = estimate for GFR since creatinine is not significantly secreted or absorbed in the renal tubules

273
Q

What level of protein on a urine dipstick warrants further testing?

  • what further testing can you order? (2 options)
  • what test can be used for tracking progression of proteinuria but not for initial diagnosis?
A

> 2+ protein = need to do further testing
-can have false positives

Further testing:
1. Protein-Cr ratio: only requires one sample but not as accurate as 24-h protein collection
2. 24 hr urine for protein = GOLD STANDARD for severity of protein loss but difficult to obtain
(Microalbumin/Cr = good for tracking progression of disease but is not used for initial diagnosis)

274
Q

What is the criteria for proteinuria?
-What is the criteria for nephrotic range proteinuria?

A
  • Proteinuria = > 0.2 urine spot protein/Cr ratio or >4 mg/m2/h on 24 hr urine collection
  • nephrotic range proteinuria = > 2.0 urine spot protein/Cr ratio or > 40 mg/m2/h on 24 hour urine collection (10x more than proteinuria) = this is NEVER benign and always warrants further evaluation
275
Q

What is the differential diagnosis for proteinuria?

  • benign causes (6)
  • pathologic causes (9)
A

Benign causes:

  1. Acute illness
  2. Fever
  3. Pregnancy
  4. Trauma
  5. Exercise
  6. Orthostatic proteinuria

Pathologic causes:

  • Glomerular:
    1. Nephrotic syndrome
    2. Glomerulonephritis
    3. Reflux nephropathy
    4. Long-standing infections
  • Tubular:
    5. Fanconi syndrome
    6. Metal poisoning
    7. Ischemic injury (ie. ATN)
  • Increased protein burden (overwhelms tubular reabsorption)
    8. Rhabdomyolysis
    9. Neoplasms
276
Q

What is the most common cause of persistent proteinuria in school-aged children and adolescents?

  • clinical features?
  • how is diagnosis made?
  • treatment?
A

Orthostatic proteinuria
-occurs in up to 60% of children with persistent proteinuria

Clinical features:

  • usually asymptomatic and condition is discovered on routine urinalysis
  • patients excrete normal or minimally increased amounts of protein in the supine position; however, in the upright position, urinary protein excretion increases up to 10 fold (1 g/24 h) once patient is up and moving.
  • Hematuria, hypertension, hypoalbuminemia, edema and renal dysfunction MUST BE ABSENT to make this diagnosis
  • diagnosis: Collect first morning urine sample (must be first voided urine immediately upon rising in the morning) = absence of proteinuria (dipstick negative or trace for protein and urine Pr:Cr
277
Q

What is the work-up for proteinuria?

  • lab tests?
  • imaging tests?
A

Initial labs: UA with microscopy, lytes, BUN, Cr, albumin, serum protein, urine protein, urine Cr to calculate Ur Pr:Cr ratio. Consider CRP, ESR, complement

  • always order renal U/S to rule out structural abnormality that might lead to obstruction of renal vessels
  • renal biopsy may or may not be indicated depending on suspected diagnosis
278
Q

What is the definition of hematuria?
-definition of gross hematuria vs. microscopic hematuria?

A

>5 RBCs/HPF on urine microscopy

  • gross hematuria = visible redness of urine with > 5 RBCs/HPF on microscopy
  • microscopic hematuria = normally colored urine with > 5 RBCs/HPF on microscopy x 2 occasions or more
279
Q

What is the differential diagnosis for hematuria?

A

Pre-renal: (not true hematuria)
1. Hemolysis = hemoglobinuria = positive heme in urine but NO RBCs
2. Rhabdomyolysis = myoglobinuria = positive myoglobin in urine but NO RBCs
***other causes of red urine but not hematuria: rifampin, chloroquine, beets, food coloring, etc.
3. Coagulopathy

Renal:

  1. IgA nephropathy
  2. Postinfectious glomerular nephritis
  3. Alport syndrome (hereditary nephritis)
  4. Thin glomerular basement membrane nephropathy
  5. Membranous nephropathy
  6. Focal segmental glomerulosclerosis

Multisystem disease:

  1. SLE nephritis
  2. HSP nephritis
  3. Wegener granulomatosis
  4. HUS

Tubulointerstitial disease:

  1. Pyelonephritis
  2. Interstitial nephritis
  3. ATN

Vascular:
14. Arterial or venous thrombosis

Malignancy:
15. Wilms, rhabdomyosarcoma, angiomyolipoma = look for abdo mass and order renal U/S

Post-renal:

  1. Trauma (look at urethra, genital lesions)
  2. Cystitis
  3. Renal calculus
280
Q

What information on the microscopy can give you a clue as to where the location of defect is causing hematuria?

A

Look at the RBC shape!

  • If normally shaped, think post-renal causes
  • if abnormally shaped RBCs, hemoglobin, RBC casts, or proteinuria, think renal parenchymal causes
281
Q

You see a patient presenting with gross hematuria. There is proteinuria and RBC casts seen on urine microscopy. Which region of the kidney is most likely involved?

A

Glomerulus!

  • will usually see brown, cola or tea colored urine
  • will see proteinuria, RBC casts and deformed urinary RBCs (acanthocytes)
282
Q

What can give false-negative results on urinalysis for a child presenting with gross hematuria?
-what can give false-positive results in a child who is not having hematuria?

A

False negatives: presence of formalin or high urinary concentrations of vitamin C

False positives: alkaline urine, contamination with hydrogen peroxide used to clean the perineum before obtaining a specimen

283
Q

What are common causes of rhabdomyolysis? (9)

A
  1. Prolonged seizures
  2. viral myositis
  3. Crush injury
  4. Severe electrolyte abnormalities (hyperNa, hypoPO4)
  5. hypotension
  6. extreme exercise
  7. DIC
  8. toxins
  9. metabolic disorders
284
Q

A patient presents to you with gross hematuria. On urine microscopy, you see RBCs, leukocytes, and renal tubular epithelial cell casts. Where is the most likely location of the renal abnormality?

A

Convoluted or collecting tubules

285
Q

What is the most common cause of gross hematuria in children?

A

Bacterial urinary tract infection

286
Q

You find microscopic hematuria on a routine UA performed for one of your patients. They are otherwise asymptomatic, physical exam is normal, and there is no personal or family history of renal disorders. What is your management plan?

A
  1. Repeat the UA and BP in one month
  2. If positive, initiate further work up at that time
287
Q

What is the most common cause of renal failure in neonates?

A

Obstruction due to outflow anomalies

288
Q

What is the differential diagnosis for acute kidney injury?

A

Prerenal:

  1. Dehydration leading to hypoperfusion
  2. Cardiogenic shock
  3. Sepsis
  4. Hemorrhage

Renal:

  1. Glomerular disease: acute glomerulonephritis (postinfectious, SLE, HSP, membranoproliferative, anti-glomerular basement membrane)
  2. Tubular disease: ATN, AIN, HUS, tumor lysis syndrome
  3. Vascular: vasculitis, renal vein thrombosis

Postrenal:
1. acute urinary obstruction = posterior urethral valves, ureterocele, tumor, urolithiasis, neurogenic bladder

289
Q

What are the clinical manifestations of acute kidney injury? (5)

A
  1. Decreased urine output
  2. Fluid overload (weight gain, peripheral edema, pulmonary edema)
  3. Hypertension (headaches, papilledema)
  4. Electrolyte abnormalities: hyperkalemia, acidosis
  5. Elevated BUN and Cr = signs of uremia (pericardial rub)
290
Q

What is the pediatric rifle criteria?

A

RIFLE = risk, injury, failure, loss, end-stage

  • Risk: estimated creatinine clearance (CCI) decreased by 25%, U/O 4 wk
  • End stage: persistent failure > 3 mo
291
Q

How do you calculate FENa?
-what does FENa 2%?

A

FENa helps you differentiate between prerenal cause of ARF from renal cause of ARF

FENa calculation:
100 x (Urinary sodium x plasma creatinine) / (plasma sodium x urinary creatinine)
-think “U PE, PE U!!” Gotta pee first to get a “peeewww, it stinks!”
-OR just remember that U pee sodium = urine sodium is on top (numerator) and then you can figure out the rest!

-Prerenal cause: patients whose urine showed an ELEVATED specific gravity (ie. concentrated), elevated urine osmolality (UOsm > 500 mOsm/kg), low urine sodium (UNa 40 mEq/L) and FENa > 2%
= kidney is not able to concentrate urine and thus losing lots of Na and fluids

292
Q

What are clinical features of acute interstitial nephritis?
-Cause?

A

AIN = immune-mediated inflammatory response affecting renal interstitium and tubules (spares glomeruli and vasculature)
-clinical features: acute renal failure, fever, flank pain, rash, arthralgias from hypersensitivity reaction and inflammation

293
Q

What drugs are known to be causes of AIN? (3 groups)

  • abnormal lab findings in patients with AIN?
  • treatment?
A
  1. Antibiotics (penicillin analogs, cephalosporins, sulfonamides, rifampin)
  2. NSAIDs
  3. Diuretics (thiazides and furosemide)
    - lab findings: urinary sediment (RBCs, EOSINOPHILS, WBC casts), proteinuria, FENa > 1%, electrolyte abnormalities
    - treatment: usually high fluid volume to dilute and wash through the offending agent
294
Q

What are main concerning clnical features of chronic renal failure?

A
  1. Anemia (decreased Epo production)
  2. Growth failure
  3. Fluid overload
  4. Bone deformities (vit D deficiency)
  5. Electrolyte abnormalities
  6. Hypertension
295
Q

What is the most common cause of nephrotic syndrome in young children

A

Minimal change disease

  • always assume that young children with nephrotic syndrome have minimal change disease and treat them with steroids - only get a renal biopsy if they do not respond to steroids
  • steroids 2 mg/kg/day x 4-6 weeks, then taper
  • most patients with minimal change disease will respond within the first month of treatment with normalization of urinary protein excretion
296
Q

Compare and contrast peritoneal dialysis vs. hemodialysis.

A

Peritoneal dialysis:

  • slow, small volume fluid shifts
  • done at home every night during sleep
  • need initial minor surgery to place peritoneal catheter
  • infection risk: peritonitis
  • slow toxin removal
  • better for growth than HD
  • children feel better with PD since there are no acute fluid shifts and less cytokine release as a result

HD:

  • faster large volume fluid shifts
  • rapid toxin removal
  • needs to be done 3x/wk at medical centre
  • need initial minor surgery to place graft
  • infection risk: graft infection/bacteremia
297
Q

What are the clinical features of nephrotic syndrome?

A

hink PALE
1. Proteinuria > 40 mg/m2/hr, Pr:Cr ratio > 2
2. hypoAlbuminemia
3. hyperLipidemia (since liver is working overtime to try to replace lost proteins )
4. Edema
***may also have hypercoagulability from loss of protein C & S (thus increasing risk of clots) and high risk of infections from loss of complement and antibodies in urine

298
Q

What is the differential diagnosis for nephrotic syndrome? (4)

A
  1. Minimal change disease
  2. Membranous nephropathy
  3. Focal segmental glomerular sclerosis
  4. Diabetic nephropathy
299
Q

What is the underlying pathophysiology behind nephrotic syndrome?

A

Podocyte effacement, vacuolation, appearance of microvilli on electron microscopy

  • unclear etiology
  • called minimal change disease because changes to kidneys are very subtle
300
Q

What is the pathophysiology of membranous nephropathy?

  • usually seen in which population?
  • gold standard for diagnosis?
  • treatment?
  • prognosis?
A

Membranous nephropathy = rare but seen in older children, autoimmune or infectious cause in about 33%

  • gold standard for diagnosis and only way to diagnosis = renal biopsy = serum IgG complex deposition leading to glomerular basement membrane damage
  • on pathology: see basement membrane spikes, granular C3 and IgG deposits
  • treatment: immunosuppression and steroids but may not change course of illness
  • prognosis: 1/3 remission spontaneously, 1/3 ongoing proteinuria, 1/3 ESRD
301
Q

What are the 2 main glomerular disease entities?

A

Nephrotic syndrome vs. Nephritic syndrome

302
Q

Which children with nephrotic syndrome should receive a renal biopsy?

A
  1. Children of any age who do NOT respond to initial course of steroids or have persistent relapses despite steroids
  2. Children > 8 years old with nephrotic syndrome (minimal change disease less likely in older children so gotta rule out FSGS, membranous nephropathy)
  3. Presence of hypertension, renal insufficiency, RBC casts = should NOT be seeing this in minimal change disease and so biopsy
303
Q

In which population do you see focal segmental glomerular sclerosis?

  • pathophysiology?
  • diagnosis?
  • treatment?
  • prognosis?
A

Usually see FSGS in patients with HIV, hep B, IV drug users, sickle cell disease

  • pathophysiology: sclerosis and scarring of glomeruli
  • diagnosis: renal biopsy
  • treatment: long-term immunosuppression and plasmaphresis
  • prognosis: poor (high rate of ESRD in kids)
304
Q

What do you see on renal biopsy of a patient with diabetic nephropathy?

A

Basement membrane damage from glucose end products
-thick basement membrane, Kimmesteil-wilson nodules

305
Q

What are clinical features seen in nephritic syndrome?

A

PHAROH

  1. Proteinuria (but less than nephrotic range)
  2. Hematuria
  3. Azotemia (elevated BUN)
  4. RBC casts
  5. Oliguria
  6. Hypertension
306
Q

What is the differential for nephritic syndrome? (10)

A

PIGS Always Pick the Most Wealthy Hogs

  1. Postinfectious glomerulonephritis
  2. IgA nephropathy
  3. Alport syndrome
  4. Membranoproliferative GN
  5. Thin basement membrane
  6. Goodpasture/anti-GBM
  7. Wegener’s
  8. HUS
  9. SLE
  10. Pauci-immune microscopic polyangitis
307
Q

Which glomerulonephritidies are associated with low complement levels? (3)

A
  1. Postinfectious glomerulonephritis (C3 will normalize 1-8 wks after presentation)
  2. Membranoproliferative GN
  3. SLE
308
Q

How can you differentiate between IgA nephropathy vs. postinfectious glomerulonephritis in terms of:

  • clinical presentation?
  • duration of hematuria in each?
  • what about on labwork?
A
  • IgA nephropathy = hematuria starts within 1-2 days after a viral URTI. Duration of hematuria = short (5 days)
  • Postinfectious GN = hematuria starts after 7-21 days after a viral URTI or impetigo skin infection (has a latency period). Duration of hematuria = long (4-6 wks)
  • labwork: IgA = NORMAL C3 levels whereas in postinfectious GN, have low C3 levels
309
Q

How do you diagnose IgA nephropathy?

  • incidence in boys vs. girls
  • what is the benefit of ordering serum IgA levels?
  • treatment?
  • prognosis?
A

Renal biopsy = see IgA deposits in glomerulus with MESANGIAL inflammation/proliferation

  • more common in boys
  • no benefit in ordering serum IgA
  • treatment: supportive (treat hypertension and proteinuria with ACE-I, may benefit from steroids)
  • prognosis: 20-40% with ESRD but MOST have benign courses. Need long term follow up
310
Q

What is thin basement membrane disease?

  • inheritance pattern?
  • treatment?
A

A cause of glomerulonephritis = presence of persistent microscopic hematuria and isolated thinning of the glomerular basement membrane

  • type 4 collagen mutation
  • familial (sporadic or AD trait)
  • treatment: none needed
311
Q

What gene mutation causes Alport syndrome?

  • inheritance pattern?
  • what do you see on renal biopsy?
A

Type 4 collagen mutation (major component of basement membranes)

  • inheritance: 85% are X-linked, rest are autosomal recessive
  • renal biopsy: mesangial proliferation, glomerular sclerosis, FOAM CELLS (lipid containing interstitial cells)
312
Q

What are clinical manifestations of Alport syndrome?

  • investigations for work-up of Alport?
  • treatment?
  • prognosis?
A
  1. Asymptomatic microscopic hematuria
    - some may have recurrent gross hematuria 1-2 d after URTI
  2. Proteinuria in boys more commonly than in girls
  3. Bilateral sensorineural hearing loss
  4. Ocular abnormalities (anterior lenticonus = extrusion of the lens into the anterior chamber is pathognomonic, corneal erosions)

Investigations:

  1. Urinalysis of patient and all 1st degree relatives
  2. Ophtho exam
  3. Hearing test
  4. Skin or Renal biopsy

Treatment: No treatment available although ACE-I can slow progression to ESRD

Prognosis: all progress to ESRD in adulthood and eventually require dialysis or renal transplant.

  • 15% of affected boys have ESRD before age 15.
  • Boys have worse prognosis than girls
313
Q

A patient presents to you with gross hematuria . You do a ophtho exam and see anterior lenticonus. What is your diagnosis?

A

Alport syndrome!
-anterior lenticonus is pathognomonic

314
Q

What is the cause of Anti-GBM glomerulonephritis?

  • what is your diagnosis if they have this AND pulmonary hemorrhage?
  • renal biopsy findings?
  • treatment?
A

Anti-GBM glomerulonephritis = autoimmune condition where IgG attacks type 4 collagen in glomerular basement membrane

  • if pulmonary involvement = Goodpasture disease
  • renal biopsy: see anti-GBM antibiodies
  • treatment: plasmapheresis and steroids
315
Q

What is the triad of HUS?
-criteria for diagnosis?

A
  1. Microangiopathic hemolytic anemia
    - need to see acute onset anemia with microangiopathic changes on peripheral blood smear (schistocytes, burr cells, helmet cells)
  2. Renal failure
    - need to see hematuria, proteinuria or elevated Cr
  3. Thrombocytopenia
    - may not see initially early in the illness

Criteria for CONFIRMED diagnosis:

  1. Lab criteria: need both ANEMIA and RENAL INJURY (thrombocytopenia is variably seen and thus not necessary for diagnosis)
  2. Acute illness that began 3 weeks after onset of an episode of acute or bloody diarrhea

****Probable diagnosis = no clear history of acute or bloody diarrhea in preceding 3 weeks but meets lab criteria

316
Q

What are potential causes of HUS in terms of large classifications? (4)

A
  1. Infection: verotoxin producing E coli, shiga toxin-producing shigella, neuraminidase-producing Strep pneumo, HIV
  2. Genetic
  3. Diseases causing microvascular injury: SLE, HELLP, antiphospholipid antibody syndrome
  4. Medication induced: cyclosporine, tacrolimus, etc.
317
Q

What environmental things can be the source of verotoxin producing E coli causing HUS?

A
  1. Undercooked meat
  2. Unpasteurized milk or apple cider
  3. Petting farms
  4. Swimming in contaminated ponds/lakes/pools
318
Q

How is the diagnosis of HUS made?
-pathogenesis?

A

Based on clinical diagnosis!!!

  • rarely need renal biopsy since the risk of biopsy is significant during active phase of disease
  • pathogenesis: toxins cause direct endothelial cell damage in the glomerulus which then activates platelets and localized thrombosis = then the RBCs being filtered through the glomerulus get sheared and you get anemia
319
Q

What differs in the clinical presentation between E.coli related HUS vs. strep pneumo related HUS?

  • which has worse prognosis?
  • how crucial is stool culture for the diagnosis of HUS?
A
  • E coli HUS = usually have diarrheal prodrome
  • strep pneumo = usually have pneumonia and empyema when they develop HUS
  • Strep pneumo has much worse prognosis (mortality 20% compared to
320
Q

What electrolyte abnormality should you monitor for in a patient with HUS?

A

Hyperkalemia due to renal failure AND hemolysis spilling K into the serum and not being able to pee it out

321
Q

What is the incidence of CNS involvement in HUS?

A

Majority of patients with HUS have some CNS involvement = most are mild (irritability, lethargy, nonspecific encephalopathic features)
-some have seizures and strokes (due to microvascular CNS thrombosis)

322
Q

What are 3 conditions on the differential diagnosis for HUS?

A

Conditions causing acute kidney failure with hemolytic anemia and thrombocytopenia:

  1. SLE
  2. Malignant hypertension
  3. Bilateral renal vein thrombosis
323
Q

What is the prognosis for HUS?

  • mortality?
  • percentage requiring dialysis during acute phase?
  • full recovery?
  • chronic renal insufficiency?
A
324
Q

Are antibiotics indicated in diarrheal associated HUS?
-what about antibiotics in strep-pneumo associated HUS?

A

NO! Antibiotics can result in increased toxin release worsening the HUS so don’t use it!
-if pneumonia associated HUS, then yes use antibiotics

325
Q

What type of clots can be seen in nephrotic syndrome? (3)

A

Nephrotic syndrome is considered a hypercoagulable condition!

  1. DVT with pulmonary embolism
  2. Renal vein thrombosis
  3. Sinus venous thrombosis (Neuro recommends CT head in any patient with nephrotic syndrome presenting with headache)
326
Q

A patient with postinfectious glomerulonephritis suddenly develops blurred vision, severe headaches, and altered mental status. What do you need to rule out?

A

Hypertensive encephalopathy!
-take BP right away

327
Q

What are 2 main complications of postinfectious glomerulonephritis?

A

***Overall, related to high blood pressure

  1. Hypertensive encephalopathy
  2. Heart failure from hypervolemia or hypertension
328
Q

What is the prognosis of postinfectious glomerulonephritis?
-how long does proteinuria/hypertension/hematuria last for?

A

Usually full recovery = acute phase resolves within 6-8 wks with normalization of proteinuria, hypertension but persistent microscopic hematuria can persist x1-2 years after initial presentation

329
Q

In a patient with suspected postinfectious glomerulonephritis, what are the indications for renal biopsy?

A
  1. Acute renal failure
  2. Nephrotic syndrome
  3. Absence of evidence of strep infection
  4. Normal complement levels
330
Q

What lab findings support a diagnosis of poststrep glomerulonephritis? (3)
-what is the treatment?

A
  1. Increasing ASOT titres
  2. Positive streptozyme or anti-DNase B
  3. Decreased C3 level
    - treatment: supportive
331
Q

What are the clinical manifestations of unilateral renal dysplasia/agenesis if not diagnosed prenatally? (3)

  • what is their expected renal function?
  • indication for nephrectomy?
A
  1. Hypertension (due to renin production by the damaged kidney)
  2. Recurrent UTIs
  3. Abdominal mass (secondary to obstruction)
    - should have normal renal function since their one kidney works well enough for both!
    - indication for nephrectomy: recurrent UTIs or severe HTN not managable by medical therapy
332
Q

What is the clinical manifestation of bilateral renal agenesis?

A

Either stillborn or die from respiratory failure
-renal agenesis = cannot produce urine in utero = oligohydramnios = pulmonary hypoplasia

333
Q

What are the clinical manifestations of autosomal recessive polycystic kidney disease? (6)

  • ultrasound findings?
  • prognosis?
  • treatment?
A

Presents either prenatally with oligohydramnios or early in infancy

  1. Abdominal mass from cysts = SMALL CYSTS
  2. Hypertension
  3. Potter facies
  4. Polyuria
  5. Polydipsia
  6. MAY see cysts in lung, pancreas, liver
  7. HEPATIC FIBROSIS = can have signs of portal hypertension
    - see big echogenic kidneys with cysts on ultrasound
    - prognosis: fast progression to ESRD in 100% of patients
    - treatment: dialysis, growth hormone, bone growth support, eventual transplant
334
Q

Why does chronic renal failure cause FTT?

A
  1. Protein loss = malnutrition
  2. Poor bone growth secondary to deficient Vit D production
  3. May be secondary to steroid therapy
  4. Possibly anemia (not great evidence supporting this)
335
Q

What are the clinical manifestations of autosomal dominant polycystic kidney disease?

  • associated findings? (2)
  • ultrasound findings?
  • what imaging test should all patients with ADPKD receive/
  • prognosis?
  • treatment?
A

Presents in adulthood and has slower, variable course than ARPKD

  1. Abdominal mass from renal cysts (big cysts)
  2. Family history
  3. UTIs
  4. Chronic renal failure
    - associated findings: hepatic cysts, cerebral aneurysms (all patients with ADPKD need head imaging to rule out aneurysms)
    - ultrasound: can be normaly initially
    - prognosis: slow progression to ESRD in middle age or later, some people don’t progress to ESRD
    - traetment: BP management with ACE-I, occasional need for transplant
336
Q

What is the pathophysiology of cystinosis?
-clinical manifestations?

A

Autosomal recessive disease caused by a lysosomal transporter mutation = leads to intracellular cystine accumulation and subsequent Fanconi syndrome from proximal tubule damage

  • cystine accumulates in kidneys, thyroid, cornea, pancreas
  • clinical manifestations:
    1. chronic renal failure (growth failure, rickets)
    2. ocular abnormalities (corneal opacities, visual impairment)
    3. Hypothyroidism
    4. Diabetes

****THERE ARE NO STONES WITH CYSTINOSIS (only with cystinuria)

337
Q

What investigations support the diagnosis of cystinosis? (4)
-treatment? (4)

A
  1. Urinalysis: elevated pH, bicarb, glucose and amino acids due to Fanconi syndrome
  2. Slit lamp exam = accumulation of cystine crystals in the cornea
  3. Serum cystine levels are normal, urine cystine levels mildly elevated
  4. Confirmation by cystine in leukocytes
    - treatment:
  5. Oral cysteamine (enzyme that cleaves cystine)
  6. Electrolyte and vit D replacement
  7. Growth hormone
  8. Renal transplant
338
Q

What are 3 endogenous nephrotoxins?

A
  1. Uric acid
  2. Myoglobin
  3. Hemoglobin
339
Q

What medication can cause nephrogenic DI?

A

Lithium

340
Q

What is the most common cause of nephrolithiasis in children?
-what is the most common type of renal calculi?

A

Idiopathic or familial hypercalciuria
-most common type of renal calculi = calcium stones

341
Q

What are struvite renal calculi associated with? (2)

A
  1. Proteus UTI
  2. Foreign body
342
Q

What investigations should you order if you are suspecfting renal calculi as a diagnosis?

A
  1. UA: may see hematuria
  2. AXR first to see if you can visualize the stone, then US or CT if xray negative (calcium stones are easily seen on xray and they are the most common type)
  3. BUN/Cr to evaluate renal function
  4. Serum calcium to look for hypercalcemia
  5. 24 hr urine collection for calcium, lytes, pH
  6. Straining the urine to obtain stones for analysis
343
Q

What is the treatment for nephrolithiasis?

  • specifically for normocalcemic hypercalciuria?
  • specifically for hypercalcemic hypercalciuria?
A
  1. 1.5-2x IV hydration
  2. Analgesia
  3. Strain urine for stones for analysis and to document stone passage
  4. Surgical removal or lithotripsy if large stone > 7 mm
  5. Treat underlying etiology
    - normocalcemic hypercalciuria: thiazide diuretics to increase tubular calcium reabsorption, mild decrease in dietary calcium, low Na, high K diet
    - hypercalcemic hypercalciuria = reduce serum calcium based on etiology
344
Q

What are causes of increased anion gap metabolic acidosis?
-normal anion gap metabolic acidosis?

A

Increased anion gap: MUDPILES

  • methanol
  • uremia
  • DKA, other causes of ketosis
  • paraldehyde
  • isoniazid, iron, inborn errors of metabolism
  • lactic acidosis
  • ethylene glycol
  • salicylate

Normal anion gap:

  • Diarrhea
  • renal tubular acidosis
345
Q

What are the 3 types of renal tubular acidosis?

  • what lab findings are expected with each?
  • what is the main treatment of all 3?
A
  1. Distal RTA (type I) = hypokalemia, hypercalciuria, nephrolithiasis and nephrocalcinosis, urine pH > 5.5 (cannot acidify their urine!!)
    - can be congenital or acquired (medications, acquired renal disease)
    - think HIGH things = high calcium in urine, high urine pH
  2. Proximal RTA (type II) = rarely present in isolation, usually part of Fanconi syndrome
    - get hypoPO4, glycosuria, aminoaciduria, urine pH
346
Q

Which type of RTA increases risk of rickets?

A

Proximal RTA! (secondary to hypophosphatemia)

347
Q

What is the differential diagnosis for enuresis?
-4 main categories (12 causes)

A

Increased urinary output:

  1. DM
  2. DI
  3. Sickle cell disease
  4. Excessive water intake

Increased bladder irritability:

  1. UTI
  2. Constipation

Structural problems:

  1. Ectopic ureter
  2. Epispadias (females)
  3. Thickened bladder wall (males)

Abnormal sphincter control:

  1. Spinal cord abnormalities
  2. Neurogenic bladder
  3. Sphincter weakness
348
Q

What is the definition of primary nocturnal enuresis?

  • most common cause?
  • most effective treatment?
  • other treatment options?
A

Night time incontinence after 6 yo with no preceding period of dryness

349
Q

What is the definition of primary diurnal enuresis?
-usual causes?

A

Lack of consistent daytime continence by age 4
-usual causes: stress incontinence, constipation

350
Q

What is the definition of:

  • hypertension
  • hypertensive urgency
  • hypertensive emergency
A
  • Hypertension: BP > 95th% on 3 separate occasions
  • hypertensive urgency: BP >99th% with headache or vomiting but no evidence of end organ damage
  • hypertensive emergency: BP > 99th% with end organ effects (encephalopathy, seizure and/or renal dysfunction)
351
Q

How do you properly measure a blood pressure?

A
  1. At rest x 3-5 mins
  2. Right arm at heart level resting on solid surface
  3. Width of cuff bladder = 40% of mid-humoral circumference
  4. Length of cuff bladder = 80% of mid-humoral circumference
  5. Inflate to 20 mm Hg above point at which radial pulse is gone
352
Q

Which is more likely in the following: primary vs secondary hypertension?

  • older children
  • younger children
A

Older children = more likely to be primary htn, especially obese or family history
-younger children = more likely to have secondary htn due to an underlying etiology

353
Q

What are 4 medications used to treat hypertensive urgency or emergency?
-side effects of each?

A

remember that for hypertensive emergency, you NEED to give IV meds

354
Q

What is the management of primary hypertension?

A

Start with trial of exercise, calorie reduction and sodium restriction before treating with medications

355
Q

What are medications used to treat chronic hypertension? (5 main classes)

A

Remember that medications are used to treat refractory primary HTN and secondary HTN

  1. ACE-Inhibitors
  2. Beta blockers
  3. Calcium channel blockers
  4. Diuretics
  5. Clonidine
356
Q

What are signs and symptoms associated with obstructive uropathies?
-5 causes of obstructive uropathy?

A
  1. Abdominal pain
  2. Abdominal mass
  3. Growth failure
  4. UTI
  5. Oliguria

Causes:

  1. Hydronephrosis
  2. Ureteropelvic junction obstruction
  3. Ureterocele
  4. Posterior urethral valves
  5. Duplicated collecting system
357
Q

What is the most common cause of abdominal mass in a newborn?

A

Multicystic dysplastic kidney disease

358
Q

What is the indication for prophylactic antibiotics in VUR?

A

Grades 4-5 VUR on VCUG

359
Q

What is chordee?
-treatment?

A

Downward curvature of erect penis due to scarring of tissue on ventral surface of penis = can be associated with hypospadias
-treatment: lysis of ventral adhesions for minor repairs, tissue graft for severe cases

360
Q

What is the most common penile anomaly?

A

Hypospadias = urethral meatus is on ventral surface of penis

361
Q

What are the clinical features of prune belly syndrome? (5)
-treatment?

A

Occurs only in males = absence of abdominal musculature

  1. Complete lack of abdominal wall muscles (thus belly looks wrinkly)
  2. Abdominal testes (cryptorchidism)
  3. GU abnormalities with dilated urinary collecting system (unobstructed)
    - 30% progress to end stage renal disease and end up needing renal transplant
  4. Clubfeet (from oligohydramnios if kidneys are not making urine)
  5. Malrotation

Treatment:

  1. Orchidopexy by 1 year
  2. Most likely ureteral implantation due to valve incompetence and reflux leading to UTIs
  3. Abdominoplasty later in life
362
Q

What is the definition of micropenis?
-what are 3 conditions that can cause micropenis?

A
363
Q

What is paraphimosis?

A

Incarceration and ischemia of the penile glans after insult or flipped retraction of the prepuce

364
Q

You are examining a baby who presents with irritability and notice a blue dot on the scrotum. What is your diagnosis?
-treatment?

A

Blue dot sign = testicular appendiceal torsion!

  • blue dot = engorged hypoxic appendage
  • treatment: need to get an ultrasound to rule out testicular torsion. If normal, then treat testicular appendiceal torsion with supportive care only since there are no complications from this condition. Give NSAIDs! Pain usually resolves within 2 weeks
365
Q

What is the usual bacterial etiology of epididymitis in adolescent males vs. children?
-useful screening test for epididymitis?

A

Adolescent males: STI
Children: E coli
-useful screening test = urinalysis to look for pyruria, bacteriuria

366
Q

What is the treatment for labial adhesions?

A

Usually none since most are asymptomatic and resolve by adolescence when estrogen levels increase

  • can use daily estrogen cream x 1 wk followed by petroleum jelly daily x 1 mo to prevent recurrence for symptomatic or persistent adhesions
  • surgical lysis option also
367
Q

What is Fanconi syndrome?

  • clinical features? (2)
  • causes? (4)
  • complications? (2)
A

Disease of proximal tubule of kidney = failure to reabsorb glucose, amino acids, uric acid, phosphate and bicarb!
-clinical features:
1. Polyuria, polydipsia and dehydration (from glycosuria)
2. Growth failure
-causes:
Inherited = 1. CYSTINOSIS (most common cause)
2. Wilson’s disease
3. Galactosemia

Acquired

  1. Medications: tetracyclines, valproic acid, etc.
    - complications: 1. Type II RTA (Proximal)
  2. Rickets due to hypophosphatemia
368
Q

What are the grades of VUR?

A
  1. Grade I = reflux of urine into an undilated ureter
  2. Grade II = reflux into ureter and collecting system without dilatation
  3. Grade III = dilatation of the ureter and collecting system without blunting of the calyces
  4. Grade 4 = blunting of the calyces
  5. Grade 5 = Even more dilatation and tortuosity of the ureter
369
Q

Orchidopexy eliminates the risk of which of the following:

a. testicular malignancy
b. decreased sperm count
c. torsion of testes
d. UTI
e. epididymitis

A

C! Undescended testis are highly mobile and increase risk of testicular torsion!
-orchidopexy improves fertility but the patient may still have reduced sperm counts

370
Q

What is a varicocele?

A

Dilatation of the pampiniform venous plexus due to valvular incompetence of the spermatic vein

371
Q

What are the symptoms of idiopathic hypercalciuria? (3)
-test to confirm diagnosis?

A
  1. Recurrent gross hematuria
  2. Persistent microscopic hematuria
  3. Dysuria or abdominal pain
    - test to confirm diagnosis: urine calcium:creatinine ratio (>0.2 is diagnostic)
372
Q

What is the underlying cause of Bartter syndrome?
-What lab findings are seen in Bartter syndrome?

A

Cause: autosomal recessive condition with defect in thick ascending limb of loop of henle
-usually present with persistent vomiting, polyuria/polydipsia

  1. Hypokalemia
  2. Hypercalciuria
  3. Metabolic Alkalosis
  4. Hyperaldosteronism
  5. Hyperreninemia
373
Q

What is the function of the proximal tubule?
What is the function of the Loop of Henle?
What is the function of the distal tubule?

A

Proximal tubule: reabsorption of

  1. 66% of NaCl
  2. 99% Glucose
  3. 99% Amino acids
  4. 99% Potassium
  5. 99% Phosphate
  6. 75% of bicarbonate
    - production of 1,25(OH)2-cholecalciferol (calcitriol)

Loop of Henle: reabsorption of
1. 25% of NaCl

Distal tubule (distal convoluted tubule and collecting ducts): reabsorption of

  1. DCT (impermeable to water) = The rest of the NaCl via Na-K exchange (this is where aldosterone acts to reabsorb Na in exchange for K going into urine)
  2. Collecting duct = responds to ADH to reabsorb water and also secretes H+
374
Q

What are clinical features of multicystic dysplastic disease?

  • monitoring?
  • prognosis?
A

Non-heritable

  1. Contralateral hydronephrosis
  2. VUR in contralateral kidney
  3. Late hypertension
  4. Wilms tumor
    - monitoring: ultrasound, renal function tests and BP annually. Only obtain VCUG if contralateral hydronephrosis is found
    - prognosis: in 50% of patients, cysts regress by age 7 but this does NOT change the risk of developing a malignancy
375
Q

What electrolyte abnormalities usually accompanies hypernatremic dehydration?

A
  1. Hypocalcemia
  2. Hyperglycemia
376
Q

What is the most common vasculitis in childhood?
-which immunogloublin is deposited into the kidneys?

A

HSP
-IgA deposited into the kidney

377
Q

What renal monitoring is required for HSP?

A
  1. Urinalysis x 6 months
  2. BP x 6 months
378
Q

What are fluid management principles for
syndrome with edema? (2)

A
  1. Fluid restriction if child is hyponatremic
  2. If dehydrated, then give albumin and lasix
379
Q

What are 3 lab findings seen with hyperaldosteronism?

A
  1. Hypernatremia
  2. Hypokalemia
  3. Metabolic alkalosis
    - remember that Na+ is coupled with HCO3- and kicks out K+
380
Q

What are 4 extrarenal manifestations of autosomal dominant polycystic kidney disease?

A
  1. Aortic root abnormalities
  2. Mitral valve prolapse
  3. Cysts on liver, ovaries, spleen
  4. Berry aneurysms in brain
  5. Increased risk of renal cell carcinoma
381
Q

Name 2 syndromes associated with renal cysts (other than PCKD).

A
  1. Von Hippel Lindau
  2. Tuberous Sclerosis
382
Q

In a patient with chronic hyponatremia, how fast can you replace their sodium?

A

No more than 0.5 meq increase per hour (no more than 12 mEQ/day)
-1 mmol Na increases serum Na by 1 mEQ

383
Q

What are the causes of:

  • hypovolemichyponatremia
  • hypervolemic hyponatremia
  • euvolemic hyponatremia
A
  • Hypovolemic hyponatremia:
    1. vomiting
    2. diarrhea
    3. burns
    4. 3rd spacing

Hypervolemic hyponatremia:

  1. Heart failure
  2. Liver failure
  3. Sepsis
  4. Nephrotic syndrome

Euvolemic hyponatremia:

  1. SIADH
  2. Water intoxication
  3. Hyperosmolar (high serum glucose)
384
Q

Newborn with 6 mm pelvicaliectasis documented on antenatal U/S. When do you do an ultrasound?

A

Any pelvicaliectasis > 5 mm requires postnatal ultrasound prior to discharge to rule out obstructive uropathy

385
Q

What antibiotic is used for prophylaxis of UTI with grade 3-5 VUR in baby > 2 mo?
-what about baby

A

Trimethoprim or Septra

386
Q

When do you stop antibiotic prophylaxis for history of UTI and VUR?

A

If UTI free > 1 yr, can stop prophylaxis antibiotics

387
Q

You have a 15 yo female with 2+ protein on routine exam. What is the FIRST line of testing you should order?

A

First morning urine analysis x 2 to rule out orthostatic proteinuria
-if this is ruled out, THEN pursue Pr:Cr ratio or 24 hr urine collection

388
Q

Name 5 nonpainful testicular mass.

A
  1. Testicular cancer
  2. Varicocele
  3. Spermatocele
  4. Hydrocele
  5. Nonincarcerated inguinal hernia
389
Q

What are the main differences between multicystic dysplastic kidney disease and polycystic kidney disease? (2)

A
  1. PCKD = inherited disorder; MCKD is not
  2. PCKD = bilateral; MCKD = unilateral (bilateral is not compatible with life)
390
Q

What is the most accurate imaging test to detect renal scarring?

A

DMSA!

  • better visualizes renal parenchymal abnormalities
  • can also diagnose pyelonephritis
391
Q

What 2 conditions are diagnosed via VCUG?

A
  1. Posterior urethral valves
  2. Vesicourethral reflux
392
Q

What 3 findings predict poor prognosis in Alport syndrome?

A
  1. Development of nephrotic syndrome
  2. Gross hematuria in childhood
  3. Prominent glomerular basement membrane thickening on renal biopsy
393
Q

A baby you examine has a single umbilical artery. There are no other abnormalities on exam. What is your next step?

A

Nothing!
-single umbilical artery only suggests renal anomaly if associated with other abN

394
Q

What findings on physical exam are associated with renal anomalies? (4)

A
  1. External ear anomalies
  2. Imperforate anus
  3. Scoliosis
  4. Single umbilical artery associated with other abnormalities on exam
395
Q

What is the treatment of SIADH?

A

Fluid restriction = 1 L/m2/day

396
Q

What features of a renal cyst would warrant further work up with CT scan?

A
  1. Septations in the cyst
  2. Calcifications in the cyst
  3. Solid component in the cyst
397
Q

When does most renal scarring occur?

A

Age 2-5 yo. After that, new scars are super rare. SO, if you see scarring from vesicourethral reflux, it must’ve happened when they were really young. Don’t need to repeat VCUG once kids are older and are UTI free even if they have residual VUR.

398
Q

What is the management of rhabdomyolysis?

A
  1. Hyperhydration (2x maintenance)
  2. Diuresis with mannitol perfusion
399
Q

What is the acute management of nephrotic syndrome?

A
  1. Lasix and albumin infusion for severe edema
  2. Volume restriction
  3. Steroids if typical picture for minimal change disease and no indications for renal biopsy
  4. May need antihypertensives
  5. Restriction of salt intake
400
Q

What should be your work-up for a patient with hematuria?

A
  1. Complete history and physical looking at blood pressure, optic discs, skin, abdomen, genitalia
  2. Confirmation of true hematuria by urine R&M
  3. Urine culture
  4. Urine calcium, protein, creatinine
  5. CBC, BUN, Cr, Calcium level, total protein and albumin
  6. Streptozyme or ASOT titres, C3, C4
  7. Renal U/S
401
Q

What is the most common cause of acute renal failure in children?

A

ATN

402
Q

What is the treatment for renal osteodystrophy secondary to CKD?

A

Renal osteodystrophy = related to phosphate retention from low GFR and diminished 1,25-dihydroxyvitamin D production in the kidney. Also get secondary parahyperthyroidism due to decreased calcium (from too much phosphate retention and thus loss of calcium into the urine)

  1. Restrict phosphate in the diet
  2. Oral phosphate binders
  3. Supplemental calcium
403
Q

What is the most common cause of hypertension in children?

A

Essential primary hypertension

404
Q

What are the components of VACTERL?

A

Vertebral anomalies
Anal atresia
Cardiac defects
TEF
Renal dysplasia
Limb defects

405
Q

What is the most common cause of abdominal mass in a newborn?

A

Renal cause = most commonly ureteropelvic junction obstruction (according to baby nelsons) OR multicystic dysplastic kidney disease (according to mama nelsons)

406
Q

What are possible causes of false positive proteinuria? (4)

A
  1. Hematuria
  2. Concentrated urine
  3. Alkalotic urine
  4. Use of antiseptic agents
407
Q

Volume loss and vital signs

A
408
Q

IBD UC vs CD

A
409
Q

An infant with cholestasis, triangular facies, and a pulmonic stenosis murmur is likely to have what syndrome?

A
  • Alagille syndrome (arteriohepatic dysplasia). Also called syndromic bile duct paucity
  • constellation of conjugated hyperbilirubinemia and cholestasis; typical triangular facies
  • butterfly vertebrae; and eye findings of posterior embryotoxon and Axenfeld anomaly or iris processes
  • pulmonic stenosis; peripheral pulmonic stenosis
410
Q

What is the natural history of an umbilical hernia?

A

Most umbilical hernias smaller than 0.5 cm spontaneously close before a patient is 2 years old.

411
Q

What is the most common cause of intestinal obstruction in young children?

A

Intussusception

412
Q

How commonly does intussusception appear with the classic findings?

A

colicky pain, vomiting, and passage of bloody stool

Most common: Ileocolic intussusception

413
Q

What are the weight status categories for children in terms of body mass index (BMI) percentile?

A
  • Underweight: < 5th percentile
  • Healthy weight: 5th to 85th percentile
  • Overweight: 85th to 95th percentile,
  • Obese: 95th to 98th percentile
  • Severely (morbidly) obese: ≥ 99th percentile
414
Q

Familial hyperlipidemia?

A

IIa is the most common

415
Q

How do marasmus and kwashiorkor differ clinically?

A

Kwashiorkor is edematous malnutrition as a result of low serum oncotic pressure. The low serum proteins result from a disproportionately low protein intake compared with the overall caloric intake. These children appear replete or fat, but they have dependent edema, hyperkeratosis, and atrophic hair and skin. They generally have severe anorexia, diarrhea, and frequent infections, and they may have cardiac failure.

Marasmus is severe nonedematous malnutrition caused by a mixed deficiency of both protein and calories. Serum protein and albumin levels are usually normal, but there is a marked decrease in muscle mass and adipose tissue. Signs are similar to those noted in hypothyroid children, with cold intolerance, listlessness, thin sparse hair, dry skin with decreased turgor, and hypotonia. Diarrhea, anorexia, vomiting, and recurrent infections may be noted.

416
Q

Chronic hypernatremia correction?

A

the serum sodium concentration should not be allowed to decrease faster than 0.5 mEq/L per hour and ideally not more than 10 to 12 mEq/L/24 hr.

417
Q

9FeNA calculation and intepretation

A
418
Q

What are four major endocrine and cardiac medications that have dramatically improved the lives and outcomes of children with CKD?

A
  • Use of erythropoietin, a hormone normally produced in the kidney, has largely eliminated the need for blood transfusions in children with CKD.
  • Use of 1,25-dihydroxycholecalciferol, an active form of vitamin D normally produced in the kidney, has dramatically prevented or treated osteodystrophy.
  • Growth hormone (GH) is not produced by the kidney. However, recombinant GH administration results in growth acceleration in growth-retarded children with CKD.
  • Inhibition of the renin-angiotensin-aldosterone system with ACE inhibitors or angiotensin receptor blockers (ARBs) has helped control hypertension and also prevented progression of glomerular fibrosis.
419
Q

A 12-year-old girl complains of a sore throat and passes painless coke-colored urine for 2 days. When you see her a few days later she has amber colored urine, microscopic hematuria, 2 + proteinuria, RBC casts, and no other complaints or clinical findings. What is the most likely cause of this presentation?

A

IgA nephropathy (IgAN)

The simultaneous occurrence of upper respiratory symptoms and gross hematuria make poststreptococcal glomerulonephritis less likely.

420
Q

Which types of glomerulonephritis are associated with hypocomplementemia?

A

Postinfectious glomerulonephritis, including poststreptococcal (ASPGN); staphylococcus in subacute bacterial endocarditis

  • Lupus nephritis
  • Membranoproliferative glomerulonephritis
  • C3 glomerulopathy
  • aHUS
421
Q

What is the usual time course for post infectious GN?

A

Symptoms and signs begin about 7 to 14 days after pharyngitis and as long as 6 weeks after a pyoderma

422
Q

A 7-year-old girl has a typical presentation of poststreptococcal glomerulonephritis with positive ASO titers and low serum C3, but over the next 3 months she has recurrent episodes of gross hematuria and her C3 remains very low. What diagnosis should be considered?

A

Recurrent gross hematuria and persistently low C3 are extremely rare in ASPGN. C3 glomerulopathy

423
Q

Most identifible cause of hematuria?

A

Hypercalciuria

424
Q

How is hypertension defined in children?

A

Prehypertension: BP readings between the 90th and 95th percentile or > 120/80 mm Hg in adolescents.

Hypertension: systolic BP and/or diastolic BP ≥ 95th percentile for age, sex, and height (on 3 repeated measurements on different [nonconsecutive] days).

Stage 1 hypertension: BP ≥ the 95th percentile and less than the 99th percentile + 5 mm Hg.

Stage 2 hypertension: BP > 99th percentile + 5 mm Hg.

425
Q

What are the indications for furosemide and albumin infusions in patients with nephrotic syndrome?

A

Indications are severe edema with incapacitating anasarca, cellulitis, skin breakdown, or respiratory embarrassment from pleural effusions

426
Q

Name two important complications of MCNS

A
  • Hypercoagulable state, which may result in sagittal sinus, cavernous sinuses, and renal veins thrombosis
  • Peritonitis caused by S. pneumoniae or E. coli
427
Q

What is the simplest way to estimate the glomerular filtration rate (eGFR) in the absence of a timed urine collection?

A

Modified Schwartz

The modified formula is [0.413 × (height in cm/serum creatinine in mg/dL)]

428
Q

Which urinary crystals are always pathologic?

A

Cystine crystals

429
Q

In what settings should renal tubular acidosis (RTA) be considered?

A

Chronic hyperchloremic metabolic acidosis with an inability to acidify the urine and a normal serum anion gap

430
Q

RTA types

A

Type 1 (distal) RTA:Inability of the distal tubule to secrete hydrogen; in the presence of significant acidosis, urine is not maximally acidified (pH < 5.5)

Type 2 (proximal) RTA:Decreased ability of the proximal tubule to reabsorb filtered HCO3 at normal plasma HCO3 concentrations

Type 4 RTA: Acquired or inherited tubular insensitivity to aldosterone or to an absence of aldosterone

431
Q

What is the urinary anion gap?

A

Na+K-Cl

432
Q

What are the indications for renal biopsy in nephrotic syndrome?

A

Macroscope hematuria

Severe HTN

Persistent Renal insufficiency

Low serum C3

Persistent proteinuria beyond 4 weeks of steriods

433
Q

Alternative to 24 urine collection to quantify proteinuria?

A

Urine protein/Cr ratio

<0.2 normal

>3 nephrotic

434
Q

What is the pneumonic for HTN?

A

MONSTER

M: MEds

Obesity

Neonatal Hx

Symptoms/Signs

Trends in the family

Endocrine

Renal

435
Q

What are the characteristics of Schwman Diamond?

A

AR, sBDS

Bone marrow failure

Panceatice dysfunction

Sketletal abN

INfections

436
Q

Which of the following usually accompanies hypernatremic dehydration? a. hypercalcemia, hypoglycemia

b. hypocalcemia, hyperglycemia
c. hypercalcemia, hyperglycemia

d. hypocalcemia, hypoglycemia

A

b. hypocalcemia, hyperglycemia

437
Q

4 yo female with persistent microhematuria on 2 urine dips. Grandfather has renal stones. What investigation would you do to prove the diagnosis (1)

A

Hypercalciuria - 24 hour urine calcium > 4mg/kg/day or urine calcium to creatinine > 0.2 for hypercalciuria

438
Q

14 year old girl with SLE. Hypertensive emergency BP 180/100. Name 3 drugs you would use to treat her hypertension and there route of administration

A
  • Labetolol (contraindicated in asthma patients)
  • Hydralazine (can lead to reflex tachycardia)
  • Nicardipine (can lead to reflex tachycardia)
  • Sodium nitroprusside
439
Q

Breastfed baby, presents with rash around his orifices. Also has glossitis, stomatitis, corneal dystrophy. What is the cause?

a. Vitamin C deficiency

b. Vitamin A deficiency
c. Vitamin B12 deficiency

d. Zinc deficiency

A

d. Zinc deficiency

440
Q

Young child with periorbital edema and diarrhea. His albumin is 12. What is your differential diagnosis (4)?

A
  • Nephrotic syndrome (most common cause for hypoalbuminemia is renal)
  • Liver disease (impaired production)
  • Malnutrition
  • Protein losing enteropathy (CMP allergy, Celiac disease, lymphangiectasia) - Inflammatory bowel disease
  • Congestive heart failure
  • Acute and chronic inflammatory conditions
  • Burns
441
Q

Name 2 risk factors for adenocarcinoma in the setting of a diagnosis of UC

A
  • Disease > 10 years
  • Onset before age 15
  • Pancolitis
  • PSC
442
Q

Adolescent with gastroesophageal reflux disease. Give 5 non pharmacologic treatments

A
  • Eat small meals, avoid eating before bed, avoid fatty, spicy, chocolate, caffeine, nicotine, peppermint, tomatoes
  • Stay upright 30 minutes after meal
  • Efficacy of sleep positioning in older children unclear, some evidence for left side position and head elevation
  • Weight loss in obese children
  • Eliminate smoke exposure (smoke reduces LES pressure)
443
Q

2 indications for fundoplication in 6 mo child with GERD.

A
  • Recurrent aspiration
  • Refractory or Barrett esophagitis
  • Reflux associated apnea
  • GERD associated FTT not responsive to medical therapy
444
Q
A
445
Q

What are the proliferative causes of glomerular causes of hematuria?

A
  1. Alport’s
  2. Thin GBM disease
  3. HUS

FSGS